Bài giảng số 4: Chứng minh bất đẳng thức bằng phương pháp dồn biến

60 31 0
Bài giảng số 4: Chứng minh bất đẳng thức bằng phương pháp dồn biến

Đang tải... (xem toàn văn)

Tài liệu hạn chế xem trước, để xem đầy đủ mời bạn chọn Tải xuống

Thông tin tài liệu

3) Chuùng toâi nghó laø caùc baïn seõ ñoàng yù raèng: neáu moät baøi toaùn ñaõ chuaån hoùa (töùc laø BÑT coù ñieàu kieän) thì noù seõ "gôïi yù" cho chuùng ta caùch doàn bieán (ph[r]

(1)

PHƯƠNG PHÁP DỒN BIẾN

Phan Thành Việt

Nội dung:

1 Giới thiệu

2 BĐT biến với cực trị đạt đối xứng Dồn biến kĩ thuật hàm số

4 BĐT biến với cực trị đạt biên BĐT biến

6 Dồn biến hàm lồi

7 Dồn biến giá trị trung bình Định lý dồn biến tổng quát Nhìn lại

10 Bài tập

1 Giới thiệu.

Các bạn thân mến, nhiều số BĐT mà ta gặp có dấu đẳng thức biến số Một ví dụ kinh điển

Ví dụ 1: (BĐT Cauchy) Cho x, y, z > x + y + z ≥ 3√3xyz

Có thể nói số lượng BĐT nhiều nhiều bạn thấy điều hiển nhiên Tất nhiên, không hẳn Tuy nhiên, trong trường hợp đẳng thức không xảy tất biến ta lại thường rơi vào trường hợp khác, tổng quát hơn: có số (thay tất cả) biến Ở chúng tơi dẫn ví dụ chứng minh phần sau

Ví dụ 2: (VMO) Cho x, y, z ∈ R, x2+ y2 + z2 = Thì

2(x + y + z) − xyz ≤ 10

(2)

Có thể nhiều bạn ngạc nhiên biết có bất đẳng thức mà dấu "=" xảy biến khác Ví dụ sau chứng minh phần sau

Ví dụ 3: (Jackgarfukel) Cho a, b, c số thực khơng âm có tối đa số Thì ta ln có:

a

a + b+ b

b + c + c

c + a

5

a + b + c

Ở đây, dấu đẳng thức xảy a = 3b > 0, c = (và dạng hốn vị). Các bạn tự hỏi giá trị chẳng hạn (3, 1, 0) có đặc biệt mà làm cho đẳng thức xảy Một cách trực giác, ta thấy dường điểm đặc biệt có biến Vì giả thiết biến không âm, nên biến cịn gọi biến có giá trị biên

Tóm lại, BĐT mà ta gặp, có trường hợp dấu "=" xảy thường gặp: trường hợp tất biến (ta gọi "cực trị đạt tâm"), tổng quát trường hợp có số biến (ta gọi "cực trị đạt có tính đối xứng"), trường hợp khác dấu "=" xảy có biến có giá trị biên (và ta gọi "cực trị đạt biên")

Phương pháp dồn biến đặt để giải BĐT có dạng Ý tưởng chung là: ta đưa trường hợp có hai biến nhau, biến có giá trị biên, số biến giảm Do BĐT đơn giản BĐT ban đầu, đặc biệt BĐT cịn biến cách khảo sát hàm biến số ta chứng minh BĐT đơn giản Chính tư tưởng giảm dần số biến nên phương pháp gọi phương pháp dồn biến

(3)

hết sức tổng quát Tinh thần xuyên suốt muốn bạn đọc cảm nhận tính tự nhiên vấn đề Qua đó, bạn lý giải "tại sao", để tự bước đường sáng tạo

*Ghi chú: Chúng tơi đánh dấu tốn theo mục Vì số lượng định lý nên không đánh dấu Chúng cố gắng ghi tên tác giả nguồn trích dẫn tất kết quan trọng, ngoại trừ kết

2 BĐT biến với cực trị đạt đối xứng.

Xin phác họa lại tư tưởng sau Bài toán có dạng f(x, y, z) ≥ với x, y, z biến số thực thỏa mãn tính chất nào Điều mong muốn có đánh giá f(x, y, z) ≥ f(t, t, z) với t đại lượng thích hợp tùy theo liên hệ x, y, z (ta sẽ gọi kĩ thuật dồn biến nhau) Sau kiểm tra

f (t, t, z) ≥ 0 để hoàn tất chứng minh Lưu ý biến

chuẩn hóa bước cuối tốn với biến

Trong mục này, xem xét ví dụ

Bài toán (BĐT Cauchy) Cho x, y, z > 0, chứng minh rằng

x + y + z ≥ 3√3xyz

Lời giải:

Vì BĐT đồng bậc nên cách chuẩn hóa ta giả sử x+y+z = 1 (*) Viết lại toán dạng f(x, y, z) ≥ với f(x, y, z) = − 27xyz Ta thấy thay x y t = x+y

2 điều kiện (*) bảo tồn (tức

vẫn có t + t + z = 1), nên ta phải xem xét thay đổi xyz.

Theo BĐT Cauchy với biến (chứng minh đơn giản) xy ≤ t2,

nên xyz ≤ t2z Vậy f(x, y, z) ≥ f(t, t, z).

Cuối để ý z = − 2t nên ta có:

f (t, t, z) = − 27t2z = − 27t2(1 − 2t) = (1 + 6t)(1 − 3t)2 ≥

(4)

*Nhận xét:

1) Có thể nhiều bạn bỡ ngỡ với cách chuẩn hóa Chúng tơi xin nói rõ: khơng có bí ẩn Nếu thích, bạn hồn tồn có thể chuẩn hóa theo cách khác, chẳng hạn giả sử xyz = chứng minh

f (x, y, z) ≥ 0 với f(x, y, z) = x + y + z − Khi bước dồn biến chứng

minh f(x, y, z) ≥ f(t, t, z) với t =xy Đề nghị bạn đọc tự lý giải

trong lời giải ta xét t = x+y

2 còn lại xét t =

xy, sau

hồn thành chứng minh theo cách

2) Bạn đọc thắc mắc: khơng cần chuẩn hóa khơng? Câu trả lời là: được! Thật vậy, hồn tồn xét tốn f(x, y, z) ≥ 0 với f(x, y, z) = x + y + z − 3xyz Khi bước dồn biến chứng minh f (x, y, z) ≥ f (t, t, z)với t = x+y2 hay t =xy Thực chất, điều

hồn tồn dễ hiểu, tương ứng BĐT có điều kiện BĐT khơng điều kiện (qua kĩ thuật chuẩn hóa)

3) Chúng nghĩ bạn đồng ý rằng: tốn chuẩn hóa (tức BĐT có điều kiện) "gợi ý" cho cách dồn biến (phải đảm bảo điều kiện), nhiên, ngược lại toán chưa chuẩn hóa (BĐT khơng điều kiện) có nhiều cách để dồn biến (nói chung, ta chọn cách dồn biến cho bảo toàn "nhiều" biểu thức BĐT - điều tương đương với chuẩn hóa cho biểu thức có dạng đơn giản nhất) Do đó, phối hợp tốt kĩ thuật chuẩn hóa dồn biến điều cần thiết Tuy nhiên, quen với điều bạn thấy khơng có khác biệt đáng kể chúng

Bài toán (BĐT Schur) Cho a, b, c ≥ 0, chứng minh rằng:

a3+ b3+ c3+ 3abc ≥ a2(b + c) + b2(c + a) + c2(a + b).

Lời giải:

Xeùt f(a, b, c) = a3

+ b3+ c3+ 3abc − a2(b + c) − b2(c + a) − c2(a + b) Đặt

t = b+c2 , ta hi voïng: f(a, b, c) ≥ f(a, t, t) Xeùt

d = f (a, b, c) − f (a, t, t) =

h

b + c −5

4a i

(b − c)2

(5)

chứng minh f(a, t, t) ≥ Nhưng BĐT tương đương với a(a − t)2 ≥ 0

nên hiển nhiên Bài toán chứng minh xong

*Nhận xét: Việc giả sử a = min{a, b, c} thủ thuật thường áp dụng để dồn biến Nhắc lại BĐT biến đối xứng ta giả sử

a ≤ b ≤ c (hoặc a ≥ b ≥ c), cịn trường hợp BĐT biến hốn vị vịng

quanh ta giả sử a = min{a, b, c} (hoặc a = max{a, b, c}).

Bài toán Cho a, b, c số thực dương có tích Chứng minh rằng:

1

a +

1

b +

1

c +

6

a + b + c ≥ 5.

Hướng dẫn:

Nếu toán ban đầu tốn quen thuộc, tốn khó Với kinh nghiệm thu từ toán 1, nghĩ tới việc dồn biến theo trung bình nhân để khai thác giả thiết tích ba số Một lời giải theo hướng bạn Yptsoi (Đài Loan) đưa lên diễn đàn Mathlinks, mà sau xin dẫn lại cách vắn tắt

Ta chứng minh f(a, b, c) ≥ f(a,bc,

bc) nếu giả sử a ≥ b ≥ c.

Tiếp theo, ta chứng minh f(a,bc,

bc) ≥ 5, hay laø

f



x2, x, x



≥ 5, với x =

bc

BĐT tương đương với (x − 1)2(2x4 + 4x3 − 4x2− x + 2) ≥ Vì biểu

thức ngoặc thứ hai dương với x > nên chứng minh hoàn tất Đẳng thức xảy a = b = c = 1.

Qua ví dụ trên, thấy cách dồn biến trung bình cộng trung bình nhân thật hữu dụng Tuy nhiên, cách dồn biến vô phong phú uyển chuyển Ví dụ sau minh họa cho điều

Bài toán 4.(Iran 1996) Chứng minh với a, b, c > thì:

(ab + bc + ca)

 1

(a + b)2 +

1 (b + c)2 +

1 (c + a)2

 ≥

4. Hướng dẫn:

(6)

là dấu "=" đạt a = b = c cịn có a = b, c → 0.

Các bạn nên thử để thấy cách dồn biến thơng thường trung bình cộng trung bình nhân dẫn đến BĐT vơ phức tạp Lời giải sau lấy từ ý thầy Trần Nam Dũng, mà nhìn kĩ bạn thấy mối tương quan, khơng tính tốn mà tư duy, kĩ thuật chuẩn hóa dồn biến, mà đề cập nhận xét 3) tốn

Vì BĐT đồng bậc nên ta giả sử ab + bc + ca = (*) Bây giờ ta hi vọng có đánh giá f(a, b, c) ≥

4 với f(a, b, c) biểu thức thứ hai của

vế trái BĐT cần chứng minh Ở t phải thỏa liên hệ (*), nghĩa là

t2+ 2tc = 1.

Bằng cách giả sử c = min{a, b, c} ta chứng minh f(a, b, c) ≥

f (t, t, c) Cuối cùng, ta kiểm tra f(t, t, c) ≥ 94 Ở bạn đọc thay c = 1−t2t2 vào BĐT để thấy:

f (t, t, c) = (1 − t

2)(1 − 3t2)2

4t2(1 + t2) ≥

Bài toán chứng minh xong!

*Nhận xét: Ở bước cuối, bạn khơng chuẩn hóa mà quay lại BĐT đồng bậc:

(t2+ 2tc)( (t + c)2 +

1 4t2) ≥

9

⇔ (t2+ 2tc)(8t2+ (t + c)2) − 9(t + c)2t2 ≥ ⇔ 2tc(t − c)2 ≥

Cuối đến với ví dụ mà cực trị khơng đạt tâm, BĐT đối xứng Các bạn thấy rằng, đường phần quan trọng dồn hai biến nhau, sau cực trị đạt tâm hay khơng khơng phải điều mấu chốt

Bài tốn (VMO) Cho x, y, z số thực thỏa mãn: x2

+ y2 + z2 = Chứng minh rằng: 2(x + y + z) − xyz ≤ 10.

Lời giải.

Đặt f(x, y, z) = 2(x + y + z) − xyz Chuùng ta hi vọng có f(x, y, z) ≥

(7)

định ý ta xét t ≥ 0.

Ta coù: d = f(x, y, z) − f(x, t, t) = 2(y + z − 2t) − x(yz − t2) Ta thaáy

ngay y + z − 2t ≤ yz − t2≤ Do để có d ≤ ta cần x ≤

Từ đó, ta giả sử x = min{x, y, z} Xét trường hợp x ≤ Khi đó ta dồn biến phải chứng minh f(x, t, t) ≤ 10 Thay

t =p(9 − x2)/2 ta coù:

g(x) = f (x, t, t) = 2x + 2p2(9 − x2) − x(9 − x2

)/2

Ta coù:

g0(x) = 3x

2

2 −

5 −

4x

18 − 2x2

Giaûi ta thấy phương trình g0(x) = 0 chỉ có nghiệm âm x = −1 Hơn

nữa g0liên tục g0(−2) > > g(0) nên suy g0 đổi dấu từ dương sang âm

khi qua điểm x = −1 Vậy ∀x ≤ g(x) ≤ g(−1) = 10 ta có điều phải chứng minh Trường hợp đẳng thức đạt x = −1, y = z = 2. Phần lại ta phải giải trường hợp x > 0, tức số x, y, z đều dương Lúc dấu BĐT thực ta cần đánh giá đơn giản không phải thông qua dồn biến Nếu x ≥ 3/4 thì

f (x, y, z) = 2(x + y + z) − xyz ≤ 2p3(x2 + y2+ z2) − (3

4)

3

= √

27 −27 64 < 10

Nếu x ≤ 3/4 thì

f (x, y, z) = 2(x + y + z) − xyz ≤ 2(p2(y2+ z2)+ 3/4) ≤= 2(18 + 3/4) < 10

Bài tốn chứng minh xong!

3 Dồn biến kó thuật hàm số.

(8)

Trong $2 thấy để chứng tỏ f(x, y, z) ≥ f(t, t, z) ta chỉ việc xét hiệu d = f(x, y, z) − f(t, t, z) tìm cách đánh giá cho d ≥ 0. Tuy nhiên, dạng BĐT đơn giản, phù hợp với biến đổi đại số Giả sử ta phải làm việc với biểu thức f có dạng, chẳng hạn, như:

f (x, y, z) = xk + yk + zk với k > cách biến đổi đại số trở nên

rất cồng kềnh phức tạp

Kĩ thuật hàm số dùng để giải trường hợp Ý tưởng chính này, chẳng hạn để chứng minh f(x, y, z) ≥ f(x, t, t) với t = (y + z)/2, ta xét hàm: g(s) = f(x, t + s, t − s) với s ≥ Sau chứng minh

g tăng với s ≥ (thông thường dùng công cụ đạo hàm tiện lợi), suy ra g(s) ≥ g(0), ∀s ≥ 0, ta thu điều mong muốn Một những

ví dụ quen thuộc với bạn dồn biến hàm lồi, nhiên quan sát kĩ thuật dồn biến bối cảnh tổng quát hơn, vấn đề hàm lồi trở lại mục sau toán với n biến.

Chúng tơi nhấn mạnh rằng, kĩ thuật khó, chứa đựng nét tinh tế phương pháp dồn biến Những ví dụ sau thể rõ vẻ đẹp sức mạnh phương pháp dồn biến

Bài toán Cho k > a, b, c số không âm có tối đa 1 số Chứng minh rằng:

( a

b + c)

k

+ ( b

c + a)

k

+ ( c

a + b)

k

≥ min{2,

2k} (∗)

Lời giải:

Tất nhiên ta cần chứng minh BĐT =

2k ⇔ k =

ln3

ln2 − (caùc

bạn suy nghĩ BĐT cho trường hợp lại dẫn đến BĐT đúng cho trường hợp tổng quát) Chú ý với k đẳng thức xảy ra hai chỗ a = b = c a = b, c = (và hoán vị).

Khơng tổng qt giả sử a + b + c = b ≥ c ≥ a Đặt

t = b+c2 và m = b−c2 , suy b = t + m, c = t − m, a = − 2t Khi vế trái

BĐT cần chứng minh là:

f (m) =

 1 − 2t

2t k

+ 

t + m

1 − t − m k

+ 

t − m

1 + m − t k

Vì c ≥ a nên 3t − ≥ m ≥ 0, vaø ≥ b + c = 2t neân

2 ≥ t ≥

Ta khảo sát f(m) miền m ∈ [0, 3t − 1] với t ∈ [1 3,

1

(9)

Ta coù:

f0(m) = k(t + m)

k−1

(1 − t − m)k+1

k(t − m)k−1

(1 + m − t)k+1

f0(m) ≥ ⇔ (t + m)

k−1

(1 − t − m)k+1

(t − m)k−1

(1 + m − t)k+1

⇔ g(m) := [ln(t − m) − ln(t + m)] − k + 1

1 − k [ln(1 − t − m) − ln(1 + m − t)] ≥ 0 Tiếp tục khảo sát g, ta có:

g0(m) = − 

1

t − m+

1

t + m



+ k + 1 1 − k



1 − t − m+ 1 + m − t

 ≥

−2t

(t − m)(t + m) +

k + 1

1 − k.

2(1 − t)

(1 − t − m)(1 + m − t) ≥ (1) Đánh giá k+1

1−k ≥ 2, để chứng minh (1) ta cần chứng minh

−t

t2− m2 +

2(1 − t)

(1 − t)2− m2 ≥ (1)

⇔ u(m) = −t + 4t2 − 3t3+ 3tm2− 2m2 ≥

Thaät vậy, u0

(m) < 0 nên u(m) ≥ u(3t − 1) = 2(3t − 1)(2t − 1)2 ≥

Vậy g(m) đồng biến suy g(m) ≥ g(0) = suy f0

(m) ≥ 0 suy

f (m) ≥ f (0) Nhớ m = b = c = t.

Cuối cùng, ta cần chứng minh h(t) := f(0) ≥ Viết lại:

h(t) =

 1 − 2t

2t k

+ 

t

1 − t k

Ta khảo sát h(t) miền t ∈ [0,1

3] Ta có:

h0(t) = 2kt

k−1

(1 − t)k+1

k

2k.

(1 − 2t)k−1

tk+1

⇔ 2k+1t2k ≤ [(1 − t)(1 − 2t)]k−1 (2)

Trong BĐT cuối, vế trái hàm đồng biến theo t vế phải hàm nghịch biến theo t, lưu ý t ≤

3 nên để chứng minh (2) ta cần:

2k+1 

1

2k

≤ [(1 − 3)(1 −

2 3)]

(10)

Bất đẳng thức đúng, nên h(t) nghịch biến, suy ra

h(t) ≥ h(1

3) =

Bài toán giải trọn vẹn!

Nhận xét: Để thấy nét đẹp toán này, xin dẫn ra số trường hợp riêng nó, thân chúng toán hay biết đến cách rộng rãi

1) Trường hợp k = 1, ta thu BĐT Netbit:

a b + c +

b c + a +

c a + b

3

Đây BĐT tiếng Một cách chứng minh "kinh điển" là:

a b + c +

b c + a+

c

a + b + =

a + b + c b + c +

a + b + c a + c +

a + b + c a + b

= (a + b + c)(

b + c+

1

c + a+

1

a + b)

≥ (a + b + c)

(b + c) + (c + a) + (a + b) =

2) Trường hợp k =

2, ta thu BĐT sau:

r

a b + c +

r

b c + a+

r

c a + b

Đây toán đẹp, trước biết đến BĐT ngược chiều với BĐT Netbit Có lời giải đơn gản, dùng BĐT

Cauchy: r

a b + c =

2a

2pa(b + c)

2a

a + b + c

3) Trường hợp k ≥

3, ta có BĐT sau:

( a

b + c)

k

+ ( a

b + c)

k

+ ( a

b + c)

k

(11)

Đây toán đẹp biết đến từ trước mở rộng cho BĐT Netbit (nó đăng tạp chí THTT với tên tác giả Trần Tuấn Anh) Từ kết tốn tổng qt, ta biết 2/3 khơng phải số tốt để có giá trị nhỏ 3/2k Tuy nhiên, số tốt theo nghĩa áp dụng BĐT Cauchy theo cách sau Để đơn giản chúng tơi trình bày với trường hợp k = 2/3.

a + b + c = a +b + c

2 +

b + c

2 ≥

3

r

a(b + c

2 )

2

⇒ ( 2a

b + c)

2

3 ≥ 3a

a + b + c

Cùng với toán 1, toán sau ví dụ đẹp cho kĩ thuật hàm số

Bài toán Cho k > 0, a, b, c ≥ a + b + c = Chứng minh rằng:

(ab)k+ (bc)k+ (ca)k ≤ max{3, (3 2)

k}

(∗)

Lời giải:

Khơng tổng qt giả sử b ≥ c (còn việc cho a = hay

max tùy theo tình huống, ta điều chỉnh cách "hợp lí" cần

thiết)

Đặt t = b+c

2 và m =

b−c

2 suy b = t + m, c = t − m Khi vế trái BĐT

cần chứng minh trở thành:

f (m) = ak[(t + m)k+ (t − m)k] + (t2− m2)k

Ta khảo sát f(m) miền m ∈ [0, t] Ta coù:

f0(m) = kak[(t + m)k−1− (t − m)k−1] − 2km(t2− m2)k−1

f0(m) ≥ ⇔ g(m) := ak[(t − m)1−k− (t + m)1−k] − 2m ≥ 0

Tất nhiên ta cần xét k > (khi k ≤ tốn đơn giản). Ta có:

(12)

⇒ g0(m) đồng biến, có tối đa nghiệm (0, t) Vì g(0) =

0, g(t) = +∞ nên có hai khả năng:

g(m) > 0 hoặc g(m) = − +

Tương ứng ta có f(m) lên f(m) xuống lại lên Trong trường hợp cực đại đạt biên

f (m) ≤ max{f (0), f (t)}

Nhắc lại m = ⇔ b = c = t vaø m = t ⇔ c = 0. Dễ thấy c = thì:

f (t) = 2(ab)k ≤ 

3

2k

nên ta phải xét trường hợp lại Đặt:

h(t) := f (0) = 2tkak+ t2k = 2tk(3 − 2t)k + t2k

Ta coù:

h0(t) = −4k(3 − 2t)k−1tk+ 2k(3 − 2t)kbk−1+ 2kb2k−1

h0(t) ≥ ⇔ −2 

3 − 2t

t

k−1

+ 

3 − 2t

t

k

+ ≥

⇔ u(x) := xk − 2xk−1+ ≥ với x = 3 − 2t

t

Ta coù: u0

(x) = [kx − 2(k − 1)]xk−2. Vì u0(x) có tối đa nghiệm R+ nên u(x) có tối đa nghiệm R+, nghiệm x = 1.

Từ đó, ta giả sử a = min{a, b, c} Khi ta việc xét t ≥ và tương ứng x ≤ Vì u(x) có tối đa nghiệm (0, 1) nên h0

(t) chỉ có tối đa nghiệm (1,3

2)

Lưu ý lưu ý h0(1) = 0, h0(3

2) > Do đó, có hai khả h(t)

đồng biến h(t) có dạng −0+ Trong trường hợp h(t) đạt max hai biên, suy ra:

h(t) ≤ max{f (1), f (3

2)} = max{3, ( 2)

2k}

(13)

*Nhận xét: Ở không giả thiết a = min{a, b, c} từ đầu là muốn nhấn mạnh rằng: việc dồn biến thực mà không cần thứ tự biến Tận dụng điều đó, làm cách khác để né việc khảo sát toán biến

Thật vậy, chứng minh ra, ta ln có BĐT sau mà khơng cần giả thiết thứ tự a, b, c:

f (a, b, c) ≤ max{(3

2)

2k

, f (a,b + c

2 ,

b + c

2 )} (∗)

Từ đó, với a, b, c cố định, xét dãy số sau: (a0, b0, c0) = (a, b, c), và

∀n ∈ Z+ ta định nghóa quy naïp:

(a2n−1, b2n−1, c2n−1) = (a2n−2,

b2n−2+ b2n−2

2 ,

b2n−2+ b2n−2

2 )

vaø:

(a2n, b2n, c2n) = (

a2n−1+ b2n−1

2 ,

a2n−1+ b2n−1

2 , c2n−1) ta có

f (a, b, c) ≤ max{(3

2)

2k

, f (an, bn, cn)}, ∀n ∈ Z+

Dễ thấy dãy {an}, {bn}, {bn} hội tụ 1, nên chuyển qua giới hạn

ta có điều phải chứng minh

Kĩ thuật chuyển qua giới hạn tự nhiên Nó tổng quát lên thành định lý dồn biến tổng quát SMV UMV mà giới thiệu phần sau Cũng sử dụng tính liên tục hàm số với kĩ thuật khác, chúng tơi cịn đạt kết tổng quát

Sau có (*), cách khác để đạt điều phải chứng minh mà cần sử dụng số hữu hạn lần thay Tuy nhiên, để khỏi trùng lắp chúng tơi giới thiệu mục BĐT biến (và mục sau), mà thực cần thiết

h Còn trường hợp biến, sử dụng cách tiếp cận đơn giản (dồn biến khảo sát), nhằm giữ tính sáng tư tưởng

(14)

thiết dồn trung bình cộng Sau ví dụ cho kiểu dồn biến trung bình nhân

Bài tốn 3: (Phạm Kim Hùng)

a) Cho số thực dương a, b, c có tích Chứng minh rằng:

(i) 81(1 + a2)(1 + b2)(1 + c2) ≤ 8(a + b + c)4 (ii) 64(1 + a3)(1 + b3)(1 + c3) ≤ (a + b + c)6

Lời giải:

(i) Đặt f(a, b, c) = 8(a + b + c)4

− 81(1 + a2)(1 + b2)(1 + c2) Ta giả sử a ≥ b Xét hàm số g(t) = f(ta, b/t, c) với t ∈ [pb/a, 1] Ta có:

g0(t) = 32(a − b

t2)(ta +

b t + c)

3

− 81(a − b

t2)(ta +

b

t)(1 + c

2

)

Vì t ∈ [pb/a, 1] nên g0(t) ≥ 0nếu:

32(d + c)3 ≥ 81d(1 + c2) với d = ta + b

t

Ta coù: 32(d+c)3

> 32d(d2+ 2dc + 3c2) ≥ 32d(3

d4c2+ 3c2

) > 81d(1 + c2) (lưu ý d2c ≥ 4)

Vaäy g0

(t) ≥ 0với t ∈ [pb/a, 1] Do đó: g(1) ≥ g(pb/a) Vậy f(a, b, c) ≥ f (s, s, c) với s =

ab Thay s = 1/cta được:

f (s, s, c) = f (√1

c,

1 √

c, c) = 8(

2 √

c+ c)

4

− 81(1 +

c)

2

(1 + c2)

= ( √

c − 1

c )

2

(8c5+ 16c92 + 24c4+ 96c

2 + 87c3+ 78c 2+

+99c2+ 120c32 − 21c + 94

c + 47)

≥ (ñpcm)

Đẳng thức xảy a = b = c = 1.

(ii) Bằng cách làm tương tự trên, bạn đọc tự chứng minh BĐT Ở xin lưu ý BĐT thực 64 số tốt

(15)

toán phức tạp Rồi thành cơng trước tốn khiến bạn tự tin Chúng dẫn tốn mà lời giải khiến nhiều bạn "khiếp sợ", nhiên chúng tơi hi vọng bạn bình tâm để thấy vẻ đẹp sáng ẩn đằng sau kĩ thuật tính tốn lão luyện

Bài tốn Cho a, b, c ≥ 0, a + b + c = Tìm giá trị lớn của biểu thức:

ab

3 + c2 +

bc

3 + a2 +

ca

3 + b2

Lời giải:

Lời giải sau anh Phan Thành Nam

Giả sử a ≥ b ≥ c Đặt a = s + t, b = s − t vế trái BĐT cần chứng minh là:

f (t) := c(s − t)

3 + (s + t)2 +

c(s + t)

3 + (s − t)2 +

s2− t2 3 + c2

Ta khảo sát f(t) miền t ∈ [0, s − c] Ta coù:

f0(t) = −c

3 + (s + t)2 −

2c(s2 − t2)

(3 + (s + t)2)2 +

c

3 + (s − t)2 +

2c(s2− t2)

(3 + (s − t)2)2 −

2t 3 + c2

= 4cst

uv +

8cst(s2− t2)(u + v)

u2v2 −

2t

3 + c2 < 0, ∀t ∈ (0, s − c) (∗)

với u = + (s + t)2, v = + (s − t)2 (BĐT (*) chứng minh sau)

Vaäy ∀t ∈ [0, s − c] thì:

f (t) ≤ f (0) = 2cs

3 + s2 +

s2

3 + c2 =

2s(3 − 2s) 3 + s2 +

s2

3 + (3 − 2s)2 =: g(s) (1)

Xét g(s) với s ∈ [1,3

2] Ta coù:

g0(s) = 24s − 12s

2

(3 + (3 − 2s)2)2+

18 − 24s − 6s2

(3 + s2)2 =

108(s2 − 3s + 4)(s − 1)2(−s2− 3s + 6)

[3 + (3 − 2s)2]2[3 + s2]2

Deã thấy s2− 3s + > 0 và −s2− 3s + = (

33−3

2 − s)(s +

33+3

2 ) neân g 0(s)

dương (1, s0) và âm (s0,32) với s0 := √

33−3

2 = 1, 372281323

Vậy ∀s ∈ [1,3 2] thì:

g(s) ≤ g(s0) =

11 √

33 − 45

(16)

Trong (1) (2), dấu "=" xảy đồng thời t = s = s0, tức

a = b = s0 vaø c = − 2s0

Vậy giá trị lớn cần tìm 11√33−45

24 = 0, 757924546 , đạt khi

a = b =

√ 33−3

2 = 1, 372281323 …, c = −

33 = 0, 255437353

Để kết thúc, ta chứng minh BĐT (*) Đây BĐT chặt Ta ra với t ∈ (0, s − c) thì:

4cs

uv <

1

3 + c2 (3) vaø

8cs(s2− t2)(u + v)

u2v2 ≤

1

3 + c2 (4)

laø xong!

Chứng minh (3): Vì c + 2s = s > nên cs < Hơn u = 3 + (s + t)2 > 4, v = + (s − t)2 > + c2 Từ suy (3)

Chứng minh (4): Dùng BĐT Cauchy ta có:

u2v2 = [[3 + (s + t)]2[3 + (s - t)]2]2 ≥ 16(s2 − t2),vaø

2cs(u + v)(3 + c2) = 4cs(3 + s2+ t2)(3 + c2) ≤ 

4cs + + s2+ t2+ + c2

3

3

Thay c = − 2s vào, lưu ý t ≤ s − c = 3s − 3, ta coù:

4cs + + s2+ t2+ + c2 ≤ 4(3 − 2s)s + + s2+ (3s − 3)2+ (3 − 2s)2 = 12 + 6(s − 1)(s − 2) ≤ 12

suy 2cs(u + v)(3 + c2

) < 43 Vaäy:

8cs(s2− t2)(u + v)

u2v2 = 4.

s2− t2

u2v2 .

2cs(u + v)(3 + c2)

3 + c2 ≤ 4.

1 44.

43

3 + c2 =

1 3 + c2

và toán giải xong!

4 BĐT biến với cực trị đạt biên.

(17)

là kó thuật dồn biến biên) Tất nhiên ta chọn s, t cho hieäu

d = f (x, y, z) ≥ f (0, s, t) đơn giản đánh giá thuận lợi Cuối

cùng ta việc kiểm chứng f(0, s, t) ≥ 0.

Trước hết, để bạn làm quen với cách dồn biến "mới mẻ" này, chúng tơi xin trở lại ví dụ phần trước

Bài toán 1: (BĐT Schur) Cho a, b, c ≥ Chứng minh rằng:

a3+ b3+ c3+ 3abc ≥ a2(b + c) + b2(c + a) + c2(a + b).

Lời giải:

Trong $2, giải cách dồn biến Tuy nhiên nhận xét điểm a = b = c, đẳng thức đạt a = b, c = 0 (và hoán vị) Do đó, kĩ thuật dồn biến biên có khả thành cơng!

Đặt f(a, b, c) = a3

+ b3+ c3+ 3abc − a2(b + c) − b2(c + a) − c2(a + b). Ta hi vọng có f(a, b, c) ≥ f(0, a + b, c) Xét hiệu:

d = f (a, b, c) − f (0, a + b, c) = ab(5c − 4a − 4b)

Như ta có d ≥ 0, cho dù tận dụng kiện a, b, c có thể

Thật đáng tiếc! Tuy nhiên, bạn dừng lại cịn đáng tiếc Thay bỏ dỡ, ta xem lại khơng thể có d ≥ Nếu tinh ý, bạn thấy f(a, b, c) nhỏ hai biến tiến lại gần (đó lý mà ta dồn hai biến $2), thay (a, b, c) (0, a + b, c) "dường như" biến cách xa Đó lý cách dồn biến thất bại

Từ đó, ta nảy ý thay (a, b, c) (0, b + a/2, c + a/2) Xét hiệu:

da= f (a, b, c) − f (0, b + a/2, c + a/2) = a(a + b − 2c)(a + c − 2b)

Điều thú vị ta giả sử da ≥ Thật vậy, điều nhờ việc

thứ tự biến a, b, c mà hiệu da, db, dc

(trong db, dc là hai hiệu tương tự da) Vì tính đối xứng nên ta

giả sử da = max{da, db, dc} Khi da < 0

(18)

và mâu thuẫn!

Vậy danên f(a, b, c) ≥ f(0, s, t) với s = b + a/2, t = c + a/2 Cuối

cùng, ta thấy

f (0, s, t) = t3+ s3− t2s − ts2 = (t + s)(t − s)2 ≥

và chứng minh hoàn tất

*Nhận xét: Mặc dù BĐT Schur quen thuộc, cách chứng minh dồn biến ý gần Tuy nhiên, cách dồn hai biến "hợp lý", cách dồn biến biên kết thực bất ngờ Tất nhiên, chứng minh cách ngắn gọn nhất, muốn nhấn mạnh đến tự nhiên

Nếu toán việc áp dụng kĩ thuật dồn biến biên gây bất ngờ, tốn sau đường tất yếu

Bài tốn 2: (Hojoo Lee) Cho a, b, c ≥ 0, ab + bc + ca = (*) Chứng minh rằng:

1

a + b +

1

b + c +

1

c + a

5 Lời giải:

Bài đẳng thức không xảy tâm, mà a = b = 1, c = các hoán vị Xét trường hợp riêng c = 0, toán trở thành:

"Chứng minh rằng:

a +

1

b +

1

a + b

5

2,với ab = 1.”

Đặt s = a+b điều tương đương với s+1

s

5

2, hay (2s−1)(s−2) ≥

0 BĐT cuối hiển nhiên s = a + b ≥ 2√ab = 2

Vậy ta cần dồn biến xong Cách làm sau lấy từ ý anh Phạm Kim Hùng Diễn Đàn Toán Học

Đặt f(a, b, c) vế trái BĐT cần chứng minh Ta hi vọng f(a, b, c) ≥

(19)

hieäu:

d = f (a, b, c) − f (a + b, a + b, 0)

=

a + b+

1

a + 1−aba+b +

1

b + 1−aba+b

!

a + b+ a + b +

1

a + b + a+b1

!

.

=

1 + a2 +

1

1 + b2 − −

1 1 + (a + b)2

Từ quy đồng lên ta thấy d ≥ 2(1 − ab) ≥ ab(a + b)2 Nếu giả sử

c = max{a, b, c} thì 2(1 − ab) = 2c(a + b) ≥ ab(a + b)2 Vậy lúc d ≥ 0

và tốn chứng minh xong!

*Nhận xét:

1) Lời giải đầy tiên đưa Diễn Đàn Toán Học anh Phan Thành Nam, cách chứng minh ngắn gọn Đặt x = a + b + c.

Nếu x ≥ thì:

a + b = c + ab

a + b ≥ c + ab

a + b + c ⇒ f (a, b, c) ≥ x +

1

x

5 Nếu x ≤ giả sử a = max{a, b, c} ta có:

f (a, b, c) = (c + ab

a + b) + (b + ac a + c) +

1

b + c

= (b + c +

b + c) +

a(1 + bc)

ax + bc ≥ +

1 =

5 (löu yù laø 2a(1 + bc) = 2a + 2abc ≥ ax + bc, x ≤ 2a ≥ )

Tuy nhiên, lời giải dễ dàng nghĩ Về lời giải dồn biến trên, lần nhấn mạnh đến tính tự nhiên

2) Bài toán toán hay thu quan tâm nhiều bạn Tuy nhiên, bạn bất ngờ hệ đơn giản BĐT quen thuộc khác Đó BĐT Iran 1996 Thật vậy, với giả thiết

ab + bc + ca = 1 từ kết BĐT Iran 1996 ta có ngay:

(

a + b +

1

b + c+

1

c + a)

2

=

(a + b)2 +

1 (b + c)2 +

1 (c + a)2 +

4(a + b + c)

(a + b)(b + c)(c + a)

4 + = 25

(20)

(lưu ý a + b + c = (a + b + c)(ab + bc + ca) ≥ (a + b)(b + c)(c + a)) Từ nhận xét trên, ta nhớ lại $2, BĐT Iran 1996 giải kĩ thuật dồn hai biến Từ có hai câu hỏi tự nhiên là, thứ nhất: tốn giải cách dồn hai biến không, thứ hai: BĐT Iran 1996 giải cách dồn biến biên không? Chúng đề nghị bạn tự giải đáp hai câu hỏi

3) Bài tốn lại dẫn đến kết thú vị sau đây, mà tác giả bạn Zhao bin (Trung Quốc)

"Cho x, y, z số thực không âm có tối đa số Chứng minh rằng:

1

x2+ y2 +

1

y2+ z2 +

1

z2+ x2 ≥

10

(x + y + z)2.”

Bằng hai tốn "cũ" trên, chúng tơi muốn bạn đọc có cảm giác dễ dàng kĩ thuật dồn biến biên Tuy nhiên, hai kĩ thuật dồn bai biến phát huy tác dụng, khơng khỏi khó khăn việc thuyết phục bạn đọc sức mạnh kĩ thuật dồn biến biên Do đó, chúng tơi dẫn toán sau đây, bạn thấy kĩ thuật dồn hai biến hoàn toàn bế tắc, đơn giản đẳng thức đạt biến đôi khác Đây một ví dụ quan trọng kĩ thuật dồn biến biên mà chúng tơi muốn trình bày với bạn

Bài toán 3: (Jackgarfukel) Cho a, b, c số thực khơng âm có tối đa số Chứng minh rằng:

a

a + b + b

b + c + c

c + a

5

a + b + c (∗)

Lời giải:

Trước công này, ta cần xem trường hợp dấu xảy ra: dễ thấy a = b = c khơng thỏa, cách tự nhiên ta nghĩ đến trường hợp biên: c = Với c = BĐT(*) trở thành

a

a + b+

b ≤

4 √

(21)

Chuaån hoùa a + b = Ta coù

(1) ⇔ − b +

b ≤

4 ⇔ ( √

b − 1/2)2 ≥ (đúng!)

Vậy đẳng thức xảy a = 3b, c = (và hoán vị).

Như trường hợp dấu xảy ba biến rời nhau, phương pháp dồn hai biến xem khơng cịn tác dụng Do đó, dồn biến biên xem đường tất yếu

Không tổng quát giả sử a = max{a, b, c} a + b + c = 1. Đặt t = a+c

2 vaø s =

a−c

2 , suy a = t + s, c = t − s, b = − 2t Ta coù

(∗) ⇔ √ t + s

s + − t +

1 − 2t

1 − t − s +

t − s

2t

5 (1)

Đặt f(s) = V T (1) với s ∈ [0, t], ta chứng minh f(s) ≤ max(f(0), f(t)). Ta có :

f0(s) =

s + − t

t + s

2(s + − t)3/2 +

1 − 2t

2(1 − t − s)3/2

1 √

2t

Vì chưa xác định dấu f0(s) nên ta đạo hàm tiếp

f00(s) = −

(s + − t)3/2 +

3(t + s) 4(s + − t)5/2 +

3(1 − 2t) 4(1 − t − s)5/2

f000(s) =

4(s + − t)5/2

15(t + s) 8(s + − t)7/2 +

15(1 − 2t) 8(1 − t − s)7/2

= 18 + 3s − 33t (1 − t − s)7/2 +

15(1 − 2t)

8(1 − t − s)7/2 > 0, b = − 2t ≥ 0

Vậy f000

(s) > 0 với s ∈ [0, t] nên theo định lí Rolle ruy f0(s)tối đa đa hai nghiệm [0, t] Mặt khác dễ dàng chứng minh f0

(0) ≤ và f0(t) ≥ 0 do f0(s) chỉ đổi dấu tối đa lần (0, t), hơn

nữa f0

(s) chỉ có dạng sau: f0(s) > 0, ∀s ∈ (0, t)

f0(s) < 0, ∀s ∈ (0, t) hoặc f0(s) có dạng − + (0, t) Tuy nhiên trong

trường hợp f(s) đạt cực đại biên.

Vậy f(s) ≤ max(f(0), f(t)) với s ∈ [0, t] nên ta cần chứng minh BĐT sau xong:

(22)

Muốn ta chứng minh BĐT f(0) ≤ 5/4 f(t) ≤ 5/4. Việc chứng minh hai BĐT dễ dàng, nên đề nghị bạn đọc tự kiểm chứng

Hẳn nhiên bạn đồng ý cần thiết phương pháp dồn biến biên tốn Tuy nhiên, nhiều bạn cho rằng: tốn khơng đối xứng nên không xảy trường hợp dấu "=" có hai biến Để phủ định nhận xét đó, chúng tơi kết thúc mục cách dẫn toán anh Phạm Kim Hùng THTT:

Bài toán Cho a, b, c ≥ 0, a + b + c = Chứng minh rằng:

(a3+ b3+ c3)(a3b3+ b3c3+ c3a3) ≤ 36(ab + bc + ca)

Lời giải:

Không tổng quát giả sử a ≥ b ≥ c Đặt

f (a, b, c) = 36(ab + bc + ca) − (a3+ b3+ c3)(a3b3+ b3+ c3+ c3a3)

Khi f(a, b + c, 0) = 36a(b + c) − (a3+ (b + c)3)a3(b + c)3

Ta chứng minh f(a, b, c) ≥ f(a, b + c, 0) Thật vậy, ý rằng:

36(ab + bc + ca) ≥ 36a(b + c)

vaø

(a3+ b3+ c3)(a3b3+ b3c3+ c3a3) ≤ [a3+ (b + c)3]a3(b + c)3 (vì ta có a3

+ b3+ c3 ≤ a3+ (b + c)3 vaø a3b3+ b3c3+ c3a3 ≤ a3(b + c)3)

Do ta cần chứng minh toán trường hợp c = 0, hay

36ab ≥ a3b3(a3+ b3) ⇔ 36 ≥ a2b2(a3 + b3)

Đặt t = ab, bất đẳng thức viết lại dạng t2(27 − 9t) ≤ 36 ⇔

t3+ ≥ 3t2 Nhưng lại BĐT Cauchy ba số t3/2, t3/2, 4 Đẳng

(23)

*Nhận xét: Một ví dụ nữa, đơn giản hơn, tác giả Diễn Đàn Toán Học:

" Cho a, b, c ≥ 0, a + b + c = Tìm giá trị lớn của:

(a2 − ab + b2)(b2 − bc + c2)(c2− ca + a2).”

Bài toán khơng khó đề nghị bạn đọc tự giải

5 BĐT biến.

Sau nắm vững kĩ thuật dồn biến với số bạn đọc mục cách nhanh chóng Chúng xin lưu ý đặc thù trường hợp biến: Khi có biến ta dồn biến theo cặp, chứng minh toán (chẳng hạn BĐT Cauchy) Tuy nhiên, thuận lợi thường xuất toán đơn giản Đối với phức tạp thường ta dồn cặp nhờ thứ tự biến Sau dồn hai biến (hoặc dồn biến biên) ta chưa có BĐT với biến, mà phải qua BĐT trung gian (2 hay biến) Tuy nhiên thường BĐT trung gian dễ để chứng minh trực tiếp đánh giá để quy biến Nói chung, nhấn mạnh điều cần thiết bạn cần quan sát thật kĩ mối liên hệ biến để có cách xử lý thích hợp

Chúng ta bắt đầu với ví dụ "kinh điển" cho kĩ thuật dồn biến với BĐT biến

Bài toán (IMO SL, Việt Nam đề nghị) Cho a, b, c, d ≥ 0, a + b + c + d = 1. Chứng minh rằng:

abc + bcd + cda + dab ≤

27 + 176

27 abcd

Lời giải:

Bài đẳng thức xảy a = b = c = d = 1/4 a = b = c = 1/3, c = Do đó, đánh giá thông thường dễ rơi vào bế tắc.

Đặt f(a, b, c, d) = abc + bcd + cda + dab − kabcd với k = 176

27 Ta có:

(24)

Từ đó, ta hi vọng có f(a, b, c, d) ≤ f(t, t, c, d) với t = a+b

2 Vì ≤ ab ≤ t

nên để có điều ta cần c + d − kcd ≥ Ở may mắn có điều có điều ngược lại, nghĩa c + d − kcd < 0, BĐT ban đầu hiển nhiên vì:

f (a, b, c, d) = ab(c+d−kcd)+cd(a+b) ≤ cd(a+b) ≤ (c + d + (a + b)

3 )

3

= 27

Vậy ta giả sử ln có f(a, b, c, d) ≤ f(a+b

2 ,

a+b

2 , c, d) Lưu ý ta

đã thực việc dồn biến mà không cần giả thiết phụ áp đặt lên biến a, b Do nhờ tính đối xứng ta dồn 2 biến biến

Từ đó, đặt thêm s = c+d

2 ta coù:

f (a, b, c, d) ≤ f (t, t, c, d) ≤ f (t, t, s, s) = f (t, s, t, s)

≤ f (t + s ,

t + s

2 , t, s) ≤ f (

t + s

2 ,

t + s

2 ,

t + s

2 ,

t + s

2 ) = f ( 4,

1 4,

1 4,

1 4) =

1 27 tốn chứng minh xong!

*Nhận xét:

1) Trong lời giải trên, thực chất bước ta lại phân trường hợp: có trường hợp dồn biến trường hợp mà BĐT hiển nhiên Do đó, lời giải khơng khỏi có phần rối rắm Bạn đọc nên trình bày lại bằng cách phản chứng (giả sử có (a0, b0, c0, d0)sao cho f(a0, b0, c0, d0) > 271)

sẽ gọn gàng chặt chẽ Một cách khác gộp hai trường hợp lại:

f (a, b, c, d) ≤ max{

27, f (

a + b

2 ,

a + b

2 , c, d)} (1)

2) Ở có cách nhìn nữa, nhìn khơng khác ý (thậm chí dài dịng hơn), nhiên kĩ thuật có ích Ý tưởng lấy từ anh Phan Thành Nam anh Phạm Kim Hùng Diễn Đàn Mathlinks

Nhắc lại f(a, b, c, d) = ab(c + d − kcd) + cd(a + b) Đặt g(x) = ab(c +

d − kcd) + cd(a + b) thì g hàm tuyến tính, ab ∈ [0, t2] (với t = a+b

2 ) neân

g(ab) ≤ max{g(0), g(t2)} Chú ý g(0) = f(0, a + b, c, d) Vậy ta có:

(25)

Với cách viết BĐT (2) việc cực trị đạt tâm biên là rõ ràng Thật ra, toán ta có f(0, a + b, c, d) ≤

27

và chuyển (2) (1) Tuy nhiên, với phức tạp dạng (2) tỏ có ích, đặc biệt kĩ thuật dồn biến tổng quát cho n số mà chúng tơi trình bày phần sau

3) Các bạn tự giải toán tương tự sau Nguyễn Anh Cường

"Giả sử x, y, z, t số thực không âm thỏa mãn x + y + z + t = 4, chứng minh rằng:

3(x2+ y2+ z2+ t2) + 4xyzt ≥ 16 ”.

Chúng ta tiếp tục với tốn mà kĩ thuật dồn biến thực rõ ràng

Bài toán (Phan Thành Nam) Cho a, b, c, d số thực khơng âm có tổng Chứng minh bằng:

abc + bcd + cda + dab + (abc)2+ (bcd)2+ (cda)2+ (dab)2≤

Lời giải:

Lời giải sau tác giả toán Đặt f(a, b, c, d) V T BĐT cần chứng minh Ta có:

f (a + b

2 ,

a + b

2 , c, d) − f (a, b, c, d)

= (a − b )

2

(c + d) + (a − b )

2

[(a + b )

2

+ ab](c2+ d2) −(a − b)

2

2 c

2

d2

≥ (a − b )

2

(c + d + 4abcd − 2c2d2)

Vậy c + d + 4abcd ≥ 2c2

d2 thì f(a+b2 ,a+b2 , c, d) ≥ f (a, b, c, d)

Ta giả sử a ≥ b ≥ c ≥ d theo ta có: f(x, x, c, d) ≥ f(a, b, c, d) với

x = a+b2 Tương tự, ta xét: f(x, x,c+d2 ,c+d2 ) − f (x, x, c, d).

Neáu 2x + 4x2cd ≥ 2x4 thì f(x, x,c+d

2 ,

c+d

2 ) ≥ f (x, x, c, d) Và ta cần

chứng minh f(x, x, y, y) ≤ với x + y = Điều đơn giản.

Nếu 2x + 4x2cd < 2x4 thì ta đánh giá tiếp: 2xcd + 2x2c2d2 <= 2x4 nên:

(26)

vaø x2(c + d) ≤ (4/3)3 neân f(x, x, c, d) <= (4/3)3 + (4/3)6 < 8 Bài

tốn chứng minh xong!

*Nhận xét:

1) Về điều kiện c + d + 4abcd ≥ 2c2d2 để dồn hai biến a, b nhau, ta

thấy cần ab ≥ cd đủ Điều có nghĩa giả sử a ≥ b ≥ c ≥ d thì ta dồn hai biến biến a, b, c (hơn nữa biến chưa BĐT thực sự, nghĩa sau dồn biến hàm f tăng lên đại lượng > 0) Liệu điều có dẫn đến:

f (a, b, c, d) ≤ f (t, t, t, c) với t = a+b+c3 hay không?

Rõ ràng, giả sử f đạt cực đại (a, b, c, d) theo ta phải có

a = b = c Trên Diễn Đàn Mathlinks bạn Zhao Bin có lời giải với ý

tưởng Tuy nhiên, việc tồn cực đại hàm f (với biến) khơng phải chuyện hiển nhiên (mặc dù rõ ràng mặc trực giác)

Một ý nữa, cách dồn biến liên tiếp biến a, b, c ta có thể dùng dãy số để chuyển qua giới hạn đưa biến Nhưng lần nữa, rõ ràng mặc trực giác cách làm không phù hợp với cách tiếp cận sơ cấp

Tuy nhiên, tốn bên chúng tơi cung cấp cho bạn cách làm thú vị để chuyển biến trường hợp

2) Nói thêm tốn Bài khơng khó theo lời tác giả tốn đặt để giải toán sau anh Phạm Kim Hùng: "Chứng minh với số không âm a, b, c, d có tổng thì:

1 5 − abc +

1 5 − bcd +

1 5 − cda +

1

5 − dab ≤ 1.”

bằng cách sử dụng bổ đề sau đây: "Cho số xi ≥ thỏa mãn: P

4

i=1(xi+ x2i) ≤ vaø xi+ xj,∀i 6= j Thì:

1 5 − x1

+

5 − x2

+

5 − x3

+

5 − x4

≤ 1.”

(27)

Trong toán sau, câu a) anh Phạm Kim Hùng, câu b) kết mạnh mà chúng tơi tìm

Bài tốn Cho a, b, c, d ≥ 0, a+b+c+d = Đặt Fk = (1 + ak)(1 + bk)(1 +

ck)(1 + dk) Chứng minh rằng:

a) F4 ≥ F3

b) F2 ≥ F1

Lời giải:

a) Ta chứng minh BĐT phản chứng Giả sử ngược lại tức tồn bốn số (a, b, c, d) thỏa mãn: a, b, c, d ≥ 0, a + b + c + d = và

F4 ≤ F3 (1)

Theo BÑT Bunhacôpski ta có: F4.F2 ≥ F32 , F3.F1 ≥ F22 , F2.F0 ≥

F2

1 (2) Từ (1) (2) suy F4 < F3< F2 < F1 < F0 = 16 (3) Từ (3) ta

coù F4 < 16 suy max(a, b, c, d) < 2.

Để dẫn tới mâu thuẫn với (3), ta chứng minh F3 ≥ F1 (4) Thật

vaäy:

(4) ⇔ (1 − a + a2)(1 − b + b2)(1 − c + c2)(1 − d + d2) ≥

⇔ (3 4+

(2a − 1)2

4 )(

3 4+

(2b − 1)2

4 )(

3 +

(2c − 1)2

4 )(

3 4+

(2d − 1)2

4 ) ≥

⇔ (1 + (2a − 1)

2

3 )(1 +

(2b − 1)2

3 )(1 +

(2c − 1)2

3 )(1 +

(2d − 1)2

3 ) ≥



4

⇔ (1 + x2)(1 + y2)(1 + z2)(1 + t2) ≥ "

1 + 

x + y + z + t

4

2#4

(5)

(Trong x = 2a−1, y =

2b−1, z =

2c−1, t =

2d−1√ )

Từ xét BĐT

(1 + A2)(1 + B2) ≥ "

1 + 

A + B

2

2#2

(6)

8(A − B)

2

(8 − A2− 6AB − B2) ≥

Ta thấy A + B ≤ BĐT đúng.

Khơng tổng qt giả sử a ≤ b ≤ c ≤ d Kết hợp với a + b +

(28)

BĐT(6) ta có:

(1 + x2)(1 + t2) ≥ "

1 + 

x + t

2

2#2

(7)

(1 + y2)(1 + z2) ≥ "

1 + 

y + z

2

2#2

(8)

nhaân (7) (8) vế theo vế suy ra:

(1 + x2)(1 + t2)(1 + y2)(1 + z2) ≥ 

1 + (x + t )

2

 

1 + (y + z )

2

2

(9)

Từ x + t < y + z < suy ra: x+t

2 +

y+z

2 < 2 Do lại áp dụng

BĐT(6) ta được: 

1 + (x + t )

2

 

1 + (y + z )

2

 ≥

" +



x + y + z + t

4

2#2

(10)

Từ (9) (10) suy ra:

(1 + x2)(1 + y2)(1 + z2)(1 + t2) ≥ "

1 + 

x + y + z + t

4

2#4

Vậy (5) suy (4) (mâu thuẫn với (3)) Điều có nghĩa việc giả sử (1) sai tức ta có BĐT ngược lại F4 ≥ F3 (đpcm)

b) Câu mạnh câu a) dùng "mánh lới" câu a khơng ổn, nhiên " đường lớn tiến cơng" khơng gặp vấn đề gì:

Đặt f(a, b, c, d) = V T − V P ta cần chứng minh f(a, b, c, d) ≥ Muốn vậy, trước hết ta chứng minh mệnh đề sau:

Mệnh đề: Nếu a + b ≤ a ≥ x ≥ b thì

f (a, b, c, d) − f (x, a + b − x, c, d) ≥ 0

Thaät vaäy:

f (a, b, c, d) − f (x, a + b − x, c, d)

= (a − x)(x − b) [(d + 1)(c + 1) − (d2+ 1)(c2+ 1)(ab − x2+ ax + bx − 2)]

(29)

Trở lại tốn ta giả sử a ≤ b ≤ c ≤ d Đặt x = a+b+c

3 thì: Chú

ý a + c ≤ c ≥ x ≥ a nên áp dụng mệnh đề ta có:

f (a, b, c, d) ≥ f (a + c − x, b, x, d) (1)

Chú ý x = (a+c−x)+b+x

3 nên x = min{x, b, a + c − x} x =

max{x, b, a+c−x}thì a+c−x = b = x neân f(a+c−x, b, x, d) = f(x, x, x, d)

và toán cịn biến

Giả sử ngược lại, có hai trường hợp:

b < x < a + c − x (2) hoặc a + c − x < x < b (3)

Lại sử dụng mệnh đề cho ta:

(2) :f (a + c − x, b, x, d) ≥ f (x, a + b + c − 2x, x, d) = f (x, x, x, d) (3) :f (a + c − x, b, x, d) ≥ f (a + b + c − 2x, x, x, d) = f (x, x, x, d) Nói chung trường hợp ta có

f (x, b, a + c − x, d) ≥ f (x, x, x, d) (2)

Từ (1) (2) suy f(a, b, c, d) ≥ f(x, x, x, d) Để giải toán biến, ta thay x = x+y+z

3 =

4−d

3 chứng minh:

f (4 − d

3 , 4 − d

3 , 4 − d

3 , d) ≥ 0 (4) Thaät vaäy:

(4) ⇔ 729(d

6

− 22d5+ 223d4− 1268d3+ 4210d2 − 7564d + 6364)(d − 1)2 ≥

Bất đẳng thức cuối nên ta có điều phải chứng minh

*Nhận xét: Việc đổi biến trước dồn biến câu a) kì lạ và đem lại hiệu khơng ngờ Kĩ thuật dồn biến câu b) mạnh, hoàn toàn sơ cấp (bởi số bước dồn biến hữu hạn) Kĩ thuật ứng dụng cực tốt cho biến Hơn thế, phần sau mở rộng để giải toán với n biến

Cuối cùng, đến với ví dụ cho trường hợp dồn biến biên Đây toán anh Phạm Kim Hùng

Bài toán Cho a, b, c, d ≥ Chứng minh rằng:

a

b2 + c2+ d2 +

b

a2+ c2+ d2 +

c

a2+ b2+ d2 +

d

a2+ b2 + c2 ≥

4

(30)

Lời giải: Xét

f (a, b, c, d) =X

4

a

b2+ c2+ d2 −

4

a + b + c + d

Giả sử a ≥ b ≥ c ≥ d Ta có:

f (a, b, c, d) − f (a, b,

a2+ b2, 0)

= c

a2+ b2 + d2 +

d

a2+ b2+ c2 −

4

a + b + c + d

−( √

c2+ d2

a2+ b2 −

4

a + b +

a2+ b2)

(do a2+ b2 ≥ c2+ d2 nên dễ thấy BĐT đúng)

Vậy vấn đề lại chứng minh f(a, b,c2+ d2, 0) ≥ 0 BĐT cuối chứng

minh khơng khó nên xin nhường lại cho bạn đọc

*Nhận xét: Cách dồn biến nhằm bảo toàn tổng a2

+ b2+ c2+ d2.

Tất nhiên, việc khơng phải điều q quan trọng, thích bạn bảo tồn a + b + c + d cách chứng minh f(a, b, c, d) ≥

f (a+b2 + a+b2 , c, d), sau đánh giá

f (t, t, c, d) ≥ 2t

t2+ (c + d)2 +

c + d

(c+d2 )2+ 2t2 −

4 2t + c + d

= x

x2/4 + y2 +

y

y2/4 + x2/2

4

x + y (trong x = 2t, y = c + d)

Bước cuối f(x, y) ≥ (chứng minh không khó bạn có thể giả sử x + y = cho gọn)

(31)

6 Doàn biến hàm lồi.

Các bạn thân mến, phương pháp dồn biến mà tìm hiểu mục trước từ trời rơi xuống Thật ý tưởng dồn biến thể rõ BĐT cổ điển Do xếp theo dịng chảy thời gian lẽ mục phải nêu từ đầu Tuy nhiên, nghĩ thú vị trở lại gốc rễ sau bạn cảm nhận dồn biến phương pháp "hiện đại"

Một cơng cụ để dồn biến BĐT "dạng cổ điển" hàm lồi Đây khái niệm quen thuộc, nhiên để tiện lợi cho bạn đọc xin nhắc lại

Định nghĩa: Một hàm số f : [a, b] → R gọi lồi nếu:

f (tx + (1 − ty)) ≤ tf (x) + (1 − t)f (y), ∀x, y ∈ [a, b], ∀t ∈ [0, 1]

*Nhận xét:

1) Nếu f khả vi lần tiêu chuẩn quan trọng để kiểm tra tính lồi là f00(x) ≥ 0, ∀x ∈ (a, b).

2) Nếu f lồi f liên tục Ngược lại, f liên tục tính lồi f là tương đương với điều "yếu hơn" là: f(x+y

2 ) ≤

f (x)+f (y)

2

Các bạn thấy, định nghĩa hàm lồi đánh vào mục tiêu dồn biến Chúng ta có kết quen thuộc sau:

Định lý: (BĐT Jensen) Cho f hàm số lồi [a, b] → R. (i) Với xi là n số thuộc [a, b] ta có:

f (x1+ x2 + + xn

n ) ≤

f (x1) + f (x2) + + f (xn)

n

(ii) Với xi là n số thuộc A λi n số khơng âm có tổng ta có:

f (λ1x1+ λ2x2+ + λnxn) ≤ λ1f (x1) + λ2f (x2) + + λ1f (xn)

(32)

Bài toán (BĐT Cauchy) Cho n số thực dương xi Chứng minh rằng:

x1+ x2+ + xn

n

n

x1x2 xn

Lời giải:

Lấy logarit vế, ta chuyển dạng:

ln(x1+ x2+ + xn

n ) ≥

ln(x1) + ln(x2) + + ln(xn)

n

Hàm số f(x) = ln(x) từ R+

→ R khả vi lần f00(x) = −x−2 < 0, ∀x >

0 Do hàm g(x) = −f(x) thỏa g00(x) > 0, ∀x > Vậy g lồi Từ đó,

áp dụng BĐT Jensen ta có điều phải chứng minh

*Nhận xét: Một cách khác thông dụng dùng để chứng minh BĐT Cauchy, đó chứng minh quy nạp theo n Cách làm hay, ta có cảm giác "cái cho n = cho n tùy ý" Các bạn hãy quan sát kĩ cách chứng minh đó, chứng minh lại BĐT Jensen, bạn thấy hàm lồi tổng quát nói lên chất vấn đề

Hàm lồi ứng dụng nhiều BĐT cổ điển, BĐT cổ diển lại giải nhiều tốn khác Tất nhiên, khơng phải công cụ "vạn năng", nhiên biết sử dụng khéo léo sức mạnh khơng nhỏ Chúng tơi dẫn ví dụ cho thấy áp dụng hàm lồi kết quả, song giúp giải trường hợp quan trọng mà trường hợp cịn lại chứng minh đơn giản cách hay cách khác

Bài toán Cho số thực x, y, z có tổng Chứng minh rằng:

x

1 + x2 +

y

1 + y2 +

z

1 + z2 ≤

9 10

Lời giải:

Xét f(t) = t

1+t2 BĐT cần chứng minh tương đương:

f (x) + f (y) + f (z) ≤ 3f (x + y + z

(33)

Do đó, −f hàm lồi coi tốn giải quyết. Ta có:

−f00(t) = 2t(3 − t

2

) (1 + t2)3

nên −f00(t) ≥ 0, ∀t ∈ [0,√3] Vậy x, y, z ∈ [0,√3]thì tốn giải

quyết

Trong trường hợp cịn lại chắn ta có dấu BĐT thực Do việc chia thành nhiều trường hợp để xét

Có thể giả sử x ≥ y ≥ z lưu ý x + y + z = x, y, z /∈ [0, 1] nên z phải âm suy f(z) < 0

*Neáu y âm suy x dương f(y) < 0, ta coù f(x) + f(y) + f(z) < f(x) < 1/2 < 9/10

*Nếu y dương suy x dương lưu ý f(y), f(x) nghịch biến [3, +∞] do f(x) + f(y) + f(z) < f(x) + f(y) < f(3) + f (

3) < 9/10 Bài toán chứng minh xong

*Nhận xét: Tất nhiên lời giải chưa phải ngắn gọn so với nhiều lời giải khác cho toán mà biết Tuy nhiên tư tưởng hồn tồn sáng Ở đây, thay mong muốn dồn biến toàn cục (dồn lần biến) việc hi vọng hợp lý dồn biến bằng lời giải ngắn Thật vậy, có biến x, y, z thuộc đoạn [0,√3]thì dùng hàm lồi ta dồn biến nhau, tốn cịn biến, xem giải xong Trong phần cịn lại việc chia trường hợp đơn giản Như vậy, có thêm kĩ thuật để dồn biến sử dụng hàm lồi

Mặc dù công cụ tốt, điểm yếu dễ nhận BĐT, biến phải nằm biểu thức độc lập (để viết thành dạng f(x1) + + f (xn)) Trong đó, BĐT mà ta gặp

phần lớn điều đó, ta phải làm việc với dạng tổng quát là f(x1, , xn) Chúng ta phải thiết lập kết dồn biến cho dạng

tổng quát mục sau

(34)

Định lý: Cho f : [a, b] → R hàm lồi Khi đó:

f (x) ≤ max{f (a), f (b)}, ∀x ∈ [a, b]

Chứng minh:

Vì f liên tục nên f đạt giá trị lớn x0 ∈ [a, b] Xét |x0− a| ≤

|x0 − b| (nghĩa x0 gần a b) Thì x1 = 2x0 − a ∈ [a, b] Khi theo

định nghóa hàm lồi ta có:

f (a) + f (x1) ≥ 2f (

a + x1

2 ) = 2f (x0)

suy f(a) = f(x0) Với x0 gần b a chứng minh tương tự

*Nhận xét: Để bạn cảm nhận "cái đúng" định lý chúng tôi nêu hình ảnh f00(x) > 0, ∀x ∈ (a, b) Khi đó, f0 đồng biến

nên có tối đa nghiệm (a, b), nói cách khác đổi dấu tối đa 1 lần Do f rơi vào trường hợp sau đây: đồng biến, nghịch biến, "đi lên xuống", "đi xuống lên" Và trường hợp ta thu kết cần thiết (Một chứng minh khác trường hợp là giả sử f đạt cực đại x0 ∈ (a, b) thì f00(x0) ≤ 0, mâu thuẫn.)

Chúng dẫn toán mà chúng thực tốn khó cho dù giải biến đổi đại số hay quy nạp

Bài toán Cho < p < q, n số thực xi∈ [p, q] Chứng minh rằng:

(x1+ x2+ + xn)(

1

x1

+

x2

+ +

xn

) ≤ n2 + 

n2

4 

(p − q)2

pq

trong kí hiệu [x] phần nghuyên x

(*Ghi chú: Đây tổng quát, trường hợp n = bài USAMO 77, n = đề thi Olympic 30 − năm 2001 )

Lời giải:

Từ giả thiết xi ∈ [p, q], ta dễ dàng đoán rằng: GTLN đạt khi

xi ∈ [p.q] với i Khi đó, g/s n số xi có k số p n − k số q thì:

V T = (kp + (n − k)q)(k q +

n − k q ) = k

2

+ (n − k)2+ k(n − k)(p

(35)

= n2+ k(n − k)(p − q)

2

pq = n

2

+1



n2− (n − 2k)2 (p − q)

2

pq

Vì k nguyên nên n2−(n−2k)2 ≤ n2(khi n chẳn) n2−(n−2k)2 ≤ n2−1

(khi n lẻ) Từ đó, ta thu BĐT ban đầu đồng thời trường hợp dấu xảy

Đến đây, ta nhận ra: mấu chốt vấn đề nhận xét: "GTLN sẽ đạt xi = p hoặc xi = q với i" Và thật bất ngờ, nhận xét

này chứng minh dễ

Với i, ta xem vế trái hàm theo xi, ta chứng tỏ: f(xi) ≤

max{f (p), f (q)} , dấu xảy xi ∈ {p, q}.

Ta coù: f(x) = Ax +B

x + C Có thể khảo sát hàm để kết (suy

ra dấu xảy xi ∈ {p, q}) Song trình bày cách sơ

cấp Để ý:

f (xi) − f (p) = (xi− p)(A −

B xip

)

f (xi) − f (q) = (xi− q)(A −

B xiq

)

Từ f(xi) > max{f (p), f (q)} thì rõ ràng xi ∈ {p, q}/ và:

A − B xip

> 0, A − B xiq

B

xip

< A < B xiq

mâu thuẫn p < q Vaäy f(xi) ≤ max{f (p), f (q)}.

Cần nói thêm trường hợp dấu bằng: g/s f(xi) = max{f (p), f (q)}

xi ∈ {p, q} Nếu f(x/ i) = f (p) thì A = xB

ip >

B

xip, f(xi) − f (p) < 0

(mâu thuẫn) Tương tự, f(xi) = f (q) cũng mâu thuẫn Vậy f(xi) =

max{f (p), f (q)} tương đương với xi ∈ {p, q}

*Nhận xét: Ta có toán mở rộng sau:

"Cho ai ∈ [a, A], bi∈ [b, B]với < a ≤ A < b ≤ B Tìm giá trị lớn

nhất cuûa

T = (a

2

1+ + a

n)(b

2

1+ + b

n)

a1b1+ + anbn

.”

Nhà toán học Polya cho chặn là:

q

AB ab +

q

ab AB

2

(36)

một điều tự nhiên đặt chặn BĐT Cơsi ? Nếu bạn tị mị xem tiếp tốn sau đây:

Bài toán (Phan Thành Nam) Cho < a < b, n số thực xi ∈ [p, q].

Chứng minh rằng:

T = x1 x2

+ x2

x3

+ +xn

x1

≤ n +hn

i (p − q)2

pq

Lời giải:

Với i, thay xi bởi p hay q trường hợp T phải tăng

lên, T không tăng buột xi ∈ {p, q}.

Cho i chạy từ tới n, với i ta thay xi bởi p hay q cho T tăng

lên (hoặc giữ nguyên hai trường hợp không tăng)

Sau bước biến đổi ta có xi ∈ [p, q]với i Nếu xi= q với

i thì T = n, khơng phải GTLN, cần xét ∃xi = p.Do hốn vị

vịng quanh nên giả sử x1 = p. Khi x3 = p hay q ta thay

x2 bởi q T không giảm Sau thay x2 bởi q ta lại thay x3 bởi p

thì T không giảm Cứ ta xen kẽ p, q số xn T

khơng giảm Sau thực trình lúc ta có

T = n

2(

p q +

q

p) (nếu n chẵn) vaø T = n − 1

2 (

p q +

q

p) + (neáu n leû)

Ta viết lại trường hợp dạng:

T = n +hn

2

i (p − q)2

pq , ∀n

và vế phải BĐT cần chứng minh Đẳng thức xảy

xi ∈ {p, q} và xen kẽ kể từ x1 tới xn (khơng kể vịng xn, x1) Bài toán đến

đây giải trọn vẹn !

(37)

biệt trường hợp cực trị đạt tâm, hàm lồi cho ta kiểu dồn biến thú vị mà tìm hiểu mục sau Mặc dù với loạt BĐT xuất gần cơng cụ cổ điển khơng đủ (hoặc khó khăn), lần nữa, nhấn mạnh tầm quan trọng ý tưởng "cổ điển", mà dựa vào "đứng vai người khổng lồ"

7 Dồn biến giá trị trung bình.

Cho đến bây giờ, phương pháp dồn biến chúng ta, số lần thực thao tác dồn biến hữu hạn, nhờ lời giải rõ ràng hoàn toàn sơ cấp Đây điều tốt mà chúng tơi muốn trì tiếp tục mục

Trước hết, giới thiệu thêm cách dồn biến dành cho hàm lồi Ta gọi kĩ thuật dồn biến giá trị trung bình, mà bạn thấy rõ điều qua kết sau:

Định lý: Cho f hàm lồi [a, b] → R Ta có:

f (a) + f (b) ≥ f (x) + f (a + b − x), ∀x ∈ [a, b]

Chứng minh:

Vì x ∈ [a, b] nên: x = ta + (1 − t)b với t ∈ [0, 1] Khi đó: a + b − x = (1 − t)a + tb Áp dụng định nghĩa hàm lồi, ta có:

f (x) + f (a + b − x) = f (ta + (1 − t)b) + f ((1 − t)a + tb)

≤ [tf (a) + (1 − t)f (b)] + [(1 − t)f (a) + tf (b)] = f (a) + f (b)

Ứng dụng kết này, ta có chứng minh cho BĐT Jensen Nhắc lại:

Định lý: (BĐT Jensen) Cho f hàm số lồi [a, b] → R Thì với xi ∈ [a, b]

là n số có trung bình cộng T, ta có:

f (x1) + f (x2) + + f (xn) ≥ nf (T )

Chứng minh:

Ta cho thực thuật tốn sau:

(38)

*Bước 2: Vì khơng có xi = T, ∀i nên phải có biến lớn hơn T

1 biến nhỏ T, mà ta giả sử x1 > T > x2 Khi thay

(x1, x2, , xn) bởi (T, x1+ x2− T, , xn) Sau trở lại bước

Như lần thực bước có trung bình cộng là T, nhiên làm cho biểu thức f tăng lên Mặt khác lần thực bước số biến T tăng lên 1, sau hữu hạn (có thể lấy n − 1) lần thực bước 2, ta phải dừng lại bước Chú ý là trình thay biểu thức f tăng lên, ta có điều phải chứng minh

Vậy có thêm cách dồn biến Sỡ dĩ không dưa cách dồn biến mục trước, có giá trị dồn biến tâm, mà với n = kĩ thuật dồn biến bằng đủ sử dụng Tuy nhiên, kĩ thuật phát huy tác dụng số biến tăng lên, cụ thể với trường hợp n biến tổng quát Lý đơn giản: BĐT với n biến, cho dù ta dồn biến chưa thu đáng kể, trường hợp sau hữu hạn lần dồn biến đưa trường hợp biến (chứ chưa nói đưa trường hợp biến nhau) Tuy nhiên, sử dụng kĩ thuật dồn biến biên dồn biến giá trị trung bình tình hình lại khác: sau lần dồn biến số lượng biến có giá trị cố định tăng lên (là giá trị biên giá trị trung bình), cần hữu hạn lần dồn biến ta đưa tất biến giá trị cố định toán xem giải xong

Tất nhiên, khả để dồn biến biên giá trị trung bình không cao Tuy nhiên, quan trọng tinh thần nó: dồn biến giá trị cố định Bạn đọc thấy ý tưởng hiệu quả trường hợp biến (xem câu c), Bài tốn 3, $5) Trong mục này, chúng tơi tiếp tục giới thiệu toán khác, mà ý tưởng dồn biến giá trị trung bình cho lời giải bất ngờ Đây toán đặc sắc anh Phạm Kim Hùng, mà việc giải chúng đem lại cho nhiều ý tưởng cho phương pháp dồn biến

Bài toán Cho n số thực dương a1, a2, , an có tích Chứng minh

rằng với k = 4(n − 1) ta ln có:

1

a1

+

a2

+ +

an

+ k

a1+ a1+ + an

≥ n + k

n (1)

(39)

Với n = 1, n = tốn đơn giản, nên ta xét n ≥ 3. Trước hết, ta khảo sát trường hợp dồn biến rút ra:

Mệnh đề 1: Kí hiệu f(a1, a2, , an) biểu thức vế trái BĐT cần chứng

minh

(i) Neáu a1 ≤ x ≤ a2 và a1a2 ≤

f (a1, a2, , an) ≥ f (x,

a1a2

x , a3, , an)

(ii) Neáu (1 − a1)(1 − a2)[ka1a2− (

Pn i=1ai)(

Pn

i=3ai+ a1a2 + 1)] ≥ 0thì

f (a1, a2, , an) ≥ f (1, a1a2, a3, , an)

(iii) Neáu a1, a2 ≥ ≥ a3 thì:

f (a1, a2, , an) ≥ min{f (1, a1a2, a3, , an), f (1, a2, a1a3, a1ai, , an)}

Chứng minh mệnh đề 1:

Để viết cho gọn ta đặt A = Pn i=3ai

(i) Ta coù:

f (a1, a2, , an) − f (x,

a1a2

x , a3, , an)

=

a1

+

a2

xx a1a2

+ k

A + a1+ a2

k

A + x + a1a2

x

= (x − a1)(a2− x)[(A + a1+ a2)(A + x +

a1a2

x ) − ka1a2]

a1a2(A + a1 + a2)(A + x + a1xa2)

Theo BÑT Cauchy:

(A + a1+ a2)(A + x +

a1a2

x )) ≥ n

2

≥ 4(n − 1) = k ≥ ka1a2

và ta có đpcm

(ii) Cũng từ đẳng thức cho x = ta có:

f (a1, a2, , an) − f (1, a1a2, a3, , an)

= (1 − a1)(1 − a2)[ka1a2− (A + a1+ a2)(A + a1a2+ 1)]

a1a2(A + a1+ a2)(A + a1a2+

và ta có đpcm

(40)

Trường hợp 1: Nếu ka1a2 ≥ (

Pn i=1ai)((

Pn

i=3ai+ a1a2+ 1)])thì dùng (ii) ta

coù f(a1, a2, , an) ≥ f (1, a1a2, a3, , an)

Trường hợp 2: Nếu ka1a2 ≤ (P

n i=1ai)((

Pn

i=3ai+a1a2+1)])thì a3 ≤ ≤ a2

nên:

ka1a3≤ (

n

X

i=1

ai)(

X

i6=1,3

ai+ a1a3+ 1)

(thaät vaäy: P

i6=1,3ai+ a1a3+

a1a3

= Pn

i=1ai− a1− a3+

a1a3

+

≥ Pn

i=1ai− a1− a2+

a1a2

+ = Pn

i=3ai+ a1a2+

a1a2

)

Do đó, dùng (ii) ta có: f(a1, a2, , ai, , an) ≥ f (1, a2, , a1ai, , an)

Mệnh đề chứng minh xong! Nó cho phép ta đưa tốn biến

Mệnh đề 2: Ta đưa tốn trường hợp có n − biến

Chứng minh mệnh đề 2:

*Bước 1: Đưa trường hợp có n − biến ≤ 1.

Giả sử có nhiều biến lớn 1, mà ta giả sử a1, a2

Thì sử dụng mệnh đề (iii) ta ln thay (a1, , an)bởi khác,

vẫn có tích 1, làm cho f khơng tăng, có số biến 1 tăng lên Do sau hữu hạn lần thay (khơng n − 1) ta sẽ có n − biến ≤ 1.

*Bước 2: Đưa n − biến ≤ nhau.

Giả sử a1 ≤ a2 ≤ ≤ an−1là n − biến có trung bình nhân là

x Nếu n − biến chưa a1 < x < an−1 dùng mệnh

đề (i) ta thay (a1, a2, , an−1, an) bởi (x, a2, ,

a1an−1

x , an) Khi

đó f khơn giảm số biến x tăng lên Ta lưu ý là

a1an−1

xa1

x ≤ 1(vì a1 là số nhỏ n − số a1, , an−1nên a1 ≤ x),

do việc thay đảm bảo n − biến ≤ 1, điều cho phép việc thay thực liên tiếp Vậy sau hữu hạn (không n−1) lần thay ta có n − biến ≤ nhau.

Cuối cùng, ta giải toán biến, tức chứng minh:

f (x, x, , x,

(41)

Đặt:

g(x) := f (x, x, , x, xn−1) =

n − 1

x + x

n−1

+ k

(n − 1)x + xn−11

với x ∈ (0, 1]. Ta có:

g0(x) = −n − 1

x2 + (n − 1)x

n−2

k[n − −

n−1 xn ]

((n − 1)x + xn−11 )2

= (n − 1)x

n− 1

x2



(n − 1)xn− 1

(n − 1)xn+ 1

2

lưu ý k = 4(n − 1)

Ta thấy g(x) ≤ với x ∈ (0, 1], nên g(x) ≥ g(1) ta có đpcm. Bài toán chứng minh xong!

*Ghi chú: Bài toán ban đầu anh Phạm Kim Hùng với k = 3n, n ≥ 4. Kết mạnh hơn, bạn thấy chứng minh cho trường hợp biến só k = 4(n − 1) "hợp lý" hơn.

Bài toán Cho n số thực dương a1, a2, , an có tích Chứng minh

raèng:

(1 + a21)(1 + a

2) (1 + a

n) ≤

2n

n2n−2(a1+ a2+ + an) 2n−2

Lời giải:

Với n = 1, n = đơn giản nên ta chứng minh cho n ≥ Ta thấy toán tương đương với f(a1, a2, , an) ≥ tương đưong với

g(a1, a2, , an) ≥ 0, đó:

f (a1, a2, , an) = k(a1+ a2+ + an)2n−2− (1 + a21)(1 + a

2) (1 + a

n)

g(a1, a2, , an) = ln(k) + (2n − 2) ln(a1+ a2+ + an)+

− ln(1 + a21) − ln(1 + a

2) − − ln(1 + a

n)

(về việc phải xét f g bình luận sau) Khảo sát sơ trường hợp dồn biến, ta có: •Mệnh đề 1:

(i) Nếu a1 ≥ ≥ a2, a3 thì:

(42)

(ii) Nếu a1 = max{ai}ni=1 và a1 ≥ x ≥ a2 ≥ thì:

g(a1, a2, , an) ≥ g(x,

a1a2

x , a3, , an)

Chứng minh mệnh đề 1: (i) Xét hiệu

f (a1, a2, , an) − f (1, a1a2, a3, , an)

= ks2n−2− ku2n−2+ [2(1 + a21a

2) − (1 + a

1)(1 + a

2)](1 + a

3) (1 + a

n)

(với s = a1+ a2 + + an, u = + a1a2+ + an)

= k(a1+ a2− − a1a2)(s2n−3+ s2n−4u + + u2n−3)+

+(1 − a21)(1 − a

2)(1 + a

3) (1 + a

n)

= −(1 − a1)(1 − a2)[k(s2n−3+ + u2n−3)+

−(1 + a1)(1 + a2)(1 + a23) (1 + a2n)]

Sử dụng lại đẳng thức với a3 đổi chỗ cho a1, ta có:

f (a1, a2, , an) − f (a1, 1, a2a3, , an)

= −(1 − a2)(1 − a3)[k(s2n−3+ + v2n−3)+

−(1 + a2)(1 + a3)(1 + a21)(1 + a

4) (1 + a

n)]

(với v = + a2a3+ a1+ a4+ + an)

Từ đẳng thức trên, ta thấy:

neáu k(s2n−3+ + u2n−3) − (1 + a1)(1 + a2)(1 + a23) (1 + a

n) ≥ (2)

Thì f(a1, a2, , an) ≥ f (1, a1a2, a3, , an)

neáu k(s2n−3+ +v2n−3)−(1+a

2)(1+a3)(1+a21)(1+a

4) (1+a

n) ≤ (3)

Thì f(a1, a2, , an) ≥ f (1, a1a2, a3, , an)

Do đó, ta cần chứng minh BĐT (2) (3) có xong! Chẳng hạn, ta giả sử (2) sai, chứng minh (3) Muốn vậy, ta cần chứng minh: u ≥ v (1+a1)(1+a23) ≤ (1+a3)(1+a21)

là xong! Điều có từ việc tính tốn đơn giản:

u − v = a3+ a1a2− a1− a2a3 = (1 − a2)(a1− a3) ≥

(1 + a1)(1 + a23) − (1 + a3)(1 + a21) = (a3− a1)(a1a3 + a1+ a3− 1) ≤

Vậy mệnh đề (i) chứng minh xong!

(43)

vào ta dùng đạo hàm Xét:

g(t) = ln(k) + 2(n − 1) ln(ta1+

a2

t + a3 + + an)+

− ln(1 + t2a21) − ln(1 +

a2

t2) − ln(1 + a

3) − − ln(1 + a

n)

với t ∈ [pa2/a1, 1]

Ta coù:

g0(t) = 2(n − 1)(a1 −

a2

t2)

ta1+ at2 + a3+ + an

2ta 1− 2a2 t3

(1 + t2a2 1)(1 +

a2

t2)

= 2(a1−

a2

t2)[

(n − 1)

ta1+ at2 + a3+ + an

ta1+

a2

t

(1 + t2a2 1)(1 +

a2

t2)

]

Vì t ∈ [pa2/a1, 1] nên a1− at2 ≥ Do đó, gọi T thừa số lại, ta

cần chứng minh T ≥ suy g đồng biến (trên [pa2/a1, 1])

Để viết cho gọn, ta đặt

c =

r

(1 + t2a2 1)(1 +

a2

t2), d = ta1+

a2

t

Ta coù:

T ≥ ⇔ n − 1

d + a3+ + an

d

c2 ⇔ (n − 1)c

≥ d2+ d(a3+ + an)

Vì c ≥ d (BĐT Bunhiacopski) nên để có BĐT ta cần:

(n − 2)c ≥ a3+ + an

Điều c > a1a2 ≥ a1 ≥ max{a3, , an}

Laáy t0 = max{x,a1xa2}/a1,

t0 ∈ [

r

a2

a1

, 1], t0a1= max{x,

a1a2

x }, a2

t0

= min{x,a1a2

x }

Vì g đồng biến [pa2/a1, 1]nên g(1) ≥ g(t0) ta có đpcm

(44)

Trở lại tốn, ta nói (a1, a2, , an) thay

(b1, b2, , bn)) nếu f(a1, a2, , an) ≥ f (b1, b2, , bn) hoặc g(a1, a2, , an) ≥

g(b1, b2, , bn)

Mệnh đề 2: Luôn đưa trường hợp có n − biến ≥ 1. Chứng minh mệnh đề 2:

*Bước 1: Đưa trường hợp có n − biến ≥ 1.

Giả sử cịn có biến a2, a3< Khi phải có biến > 1, mà ta có thể

giả sử a1 Sử dụng mệnh đề (i), ta thay (a1, a2, , an)

(1, a1a2, a3, , an) hoặc (a1, 1, a2a3, , an) Chú ý cho dù thay

nào, số biến tăng lên Do đó, động tác thay thế phải dừng lại sau không n − lần Khi đó, ta có n − 1 biến ≥

*Bước 2: Ta chứng minh ln thay n − biến ≥ trung bình nhân chúng Thật vậy, giả sử a1 ≥ a2 ≥ ≥ an−1 ≥ ≥ an

đặt x = n−1√a

1, a2, , an−1 ≥ Nếu n − biến cịn

biến khác x a1 > x > an−1 Sử dụng mệnh đề (ii) ta thay bộ

(a1, a2, , an−1, an) bởi (x, a3, ,a1xa2, an) Chú ý a1an−1

x ≥ an−1

(vì a1 là số lớn số {ai}n−1i=1 nên a1 ≥ x) việc thay

thế đảm bảo n − biến ≥ (để thay liên tiếp) Chú ý sau thay số biến x tăng lên 1. Do đó, sau khơng q n−1 lần thay n−1 biến x.

Cuối ta giải toán biến Xét hàm số h(x) := g(x, x, , x,

xn−1)

= ln(k) + 2(n − 1) ln((n − 1)x +

xn−1) − (n − 1) ln(1 + x

2

) − ln(1 +

x2n−2)

với x ≥ 1. Ta có:

h0(x) = 2(n − 1) n − −

n−1 xn

(n − 1)x + xn−11

2(n − 1)x 1 + x2 −

2(n−1)

x2n−1

1 + x2n−21

= 2(n − 1)

x



(n − 1)(xn− 1) (n − 1)xn+ 1 −

x2

1 + x2 +

1 1 + x2n−2



= 2(n − 1)

x



(n − 1)(xn− 1)

(n − 1)xn+ 1 −

x2n− 1

(1 + x2)(1 + x2n−2)

(45)

Chú ý x ≥ nên để có h0(x) ≥ 0 ta cần:

n − 1

(n − 1)xn+ 1 ≥

xn+ (1 + x2)(1 + x2n−2)

Ta đạt điều đánh giá đơn giản:

n − 1

(n − 1)xn+ 1 ≥

1

xn+ 1 ≥

xn+ (1 + x2)(1 + x2n−2)

(Có dấu ≥ thứ hai BĐT Bunhiacopski )

Vậy với x ≥ h0(x) ≥ 0 nên h(x) đồng biến, suy h(x) ≥ h(1) = và

ta có đpcm

Vậy tốn chứng minh xong! Đẳng thức xảy a1 = a2=

= an= với n ≥ (còn với n = 1, n = có đẳng thức).

*Nhận xét:

1) Bài toán anh Phạm Kim Hùng đặt dạng toán mở chứng minh chúng tơi chứng minh cho

2) Ở việc xét đồng thời hàm f, g cho phép ta mở rộng khả dồn biến: xét f đơn giản hơn, xét g đơn giản Trong tốn 1 vấn đề đơn giản nên cần hàm f đủ.

8 Định lý dồn biến tổng quát.

Các bạn thân mến, nói định lý dồn biến phải nhắc tới kết ấn tượng, định lý dồn biến mạnh (SMV) anh Phạm Kim Hùng định lý dồn biến không xác định (UMV) bạn Đinh Ngọc An Trong đó, "xương sống" định lý bổ đề dãy số, kết cho ta cảm giác rõ ràng dồn biến

Trong mục này, cung cấp cho bạn định lý dồn biến tổng quát − định lý GMV anh Phan Thành Nam − với cách tiếp cận Có thể trình bày ngắn gọn cách dẫn định lý chứng minh nó, nhiên chúng tơi khơng làm muốn chia với bạn đường (trong tư duy) để xây dựng Hi vọng sau xem xong, bạn có cảm giác có vơ số định lý dồn biến

(46)

Định nghóa 1:

Khơng gian Rn là tập hợp thứ tự x = (x

1, x2, , xn) với xi ∈ R, ∀i.

Một dãy {xm= (x1,m, , xn,m)} trong Rn gọi hội tụ z = (z1, , zn) ∈

Rn nếu dãy xi,m hội tụ zi khi m → ∞, ∀i = 1, 2, , n.

Cho D ⊂ Rn Một hàm số f : D → R gọi liên tục D nếu: với mọi

dãy {xm} ⊂ D và với z ∈ D cho {xm} hội tụ z, ta có:

f (xm) hội tụvề f(z).

Định nghóa 2: Cho D ⊂ Rn Ta nói:

D đóng với dãy {xm} ⊂ D và với z ∈ Rn sao cho {xm} hội

tụ z, ta có z ∈ D.

D bị chặn tồn số thực M cho: ∀x = (x1, , xn) ∈ D}, thì

|xi| ≤ M, ∀i = 1, 2, , n.

Ví dụ tập hợp hữu hạn đóng bị chặn Xuất phát điểm kết tuyệt đẹp sau đây:

Định lý 1: Cho D đóng bị chặn Rn, f : D → R liên tục.

Thì f đạt giá trị nhỏ D, nghĩa tồn x0 ∈ D cho:

f (x0) ≤ f (x), ∀x ∈ R.

Đây kết có chương trình phổ thơng nước, nhiên nước ta xem thuộc "Tốn cao cấp" Tuy nhiên, để tiện lợi cho bạn đọc dẫn chứng minh mà bạn hồn tồn hiểu với kiến thức phổ thơng

Chúng tơi nhắc lại kết có SGK: " dãy số thực đơn điệu bị chặn hội tụ" "Tiên đề" sử dụng để chứng minh kết dãy

Định nghĩa 2: Cho dãy số {am}∞m=1 (trong R R

n) Một dãy

{amk}

k=1 được gọi dãy dãy {am}∞m=1 nếu {mk}∞k=1 dãy

tăng ngặt số nguyên dương

*Ví dụ: {a2m}∞m=1 là dãy dãy {am}∞m=1 Dưới cận

của số bỏ qua không gây hiểu lầm

Bổ đề 1: (Weierstrass) Mỗi dãy am bị chặn R có dãy hội tụ.

Chứng minh:

Ta chứng minh có dãy đơn điệu xong Xét tập T := {m ∈ Z+|∃m0 > m sao cho a

(47)

dãy {am} giảm kể từ số Nếu T vơ hạn ta trích

1 dãy tăng Trong hai trường hợp ta ln có dãy đơn điệu

Bổ đề 2: (Weierstrass) Mỗi dãy am bị chặn Rn có dãy hội tụ

Chứng minh:

Xét {am = (x1,m, , xn,m)} là dãy bị chặn Rn Khi dãy

{x1,m} bị chặn R nên có dãy {x1,mk1} hội tụ Dãy {x2,mk1}

cũng bị chặn R nên có dãy {x2,mk2} hội tụ Bằng cách lấy

"dãy dãy con" liên tiếp vậy, cuối ta thu dãy {amk = (x1,mk, , xn,mk}mà ∀i = 1, 2, , n, ta có dãy {xi,mk}hội tụ R

Điều có nghĩa dãy {amk} hội tụ R

n

Bổ đề 3: (Tính đầy đủ R) Cho A tập bị chặn R Thì tồn tại

M ∈ R sao cho: M ≤ A (nghóa M ≤ a, ∀a ∈ A) có dãy {ak} A

hội tụ M Ta kí hiệu M = infA. Chứng minh:

Ta chứng minh ∀ε > 0, ∃a ∈ A, a − ε ≤ A Giả sử ngược lại Khi đó lấy x1 ∈ A tùy ý, quy nạp ta xây dựng dãy {xm} A

cho xm+1 ≤ xm− ε, ∀m ∈ Z+ Khi ta có: xm ≤ x1− (m − 1)ε, ∀m ∈ Z+

và điều mâu thuẫn với A bị chận Như vậy, ∀m ∈ Z+, tồn a

m ∈ A sao cho amm1 ≤ A Vì dãy {am}bị

chặn nên có dãy {amk} hội tụ M R Ta chứng minh M ≤ A

nữa xong Thật vậy, lấy a ∈ A amk −

1

mk ≤ a, ∀k ∈ Z

+, neân

cho k → ∞ suy M ≤ a.

Chứng minh định lý 1: Xét A = f(D) Ta chứng minh A có phần tử nhỏ

Ta A có tính chất sau: dãy {am} chứa A am → α

thì α ∈ A Thật vậy, theo định nghóa ta có xm∈ D sao cho f(xm) = am→ α.

Vì dãy {xm}bị chặn (chứa D) nên có dãy {xmk} hội tụ c trong

Rn Vì D đóng nên c ∈ D Vì f(xm) → α nên Vì f(xmk) → α Mặt khác,

vì {xmk} → c và f liên tục nên f(xmk) → f (c) Vì giới hạn nhất

neân f(c) = α.

(48)

Định lý mở rộng kết quen thuộc có SGK: "Cho [a, b] khoảng đóng R f : [a, b] → R liên tục, f có giá trị nhỏ [a, b]" Do đó, mặt trực giác định lý rõ ràng. Tuy nhiên, bạn khó hình dung định lý liên quan đến vấn đề dồn biến? Hệ sau định lý "chìa khóa" cho định lý dồn biến Lưu ý tất kết mục khơng cần điều kiện f đối xứng.

Định lý 2: Cho:

D tập đóng, bị chận Rn, Λ tập đóng D.

• T : D → D phép biến đổi

• f : D → R là hàm số liên tục thỏa mãn f(x) > f(T (x)), ∀x ∈ D\Λ. Thì ta có GTNN f đạt Λ, nghĩa là:

f (x) > min

y∈Λ{f (y)}, ∀x ∈ D\Λ.

Chứng minh:

Do định lý 1, tồn x0 ∈ D sao cho f(x0) ≤ f (x), ∀x ∈ D Nếu x0

không thuộc Λ f(x0) > f (T (x0)), mâu thuẫn Vậy x0 ∈ Λ ta có điều

phải chứng minh

*Ghi chú: Ta thấy phép biến đổi T : D → D thực định lý trên địi hỏi tính chất T D\Λ Do với x ∈ Λ T (x) có thể lấy giá trị tùy ý ta xem T (x) = x Quy ước được sử dụng phần lại, nghĩa T (x) = x, ∀x ∈ Λ ta quan tâm giá trị T D\Λ.

Đây hệ đơn giản phải không bạn, nhiên ý tưởng dồn biến lộ rõ Để minh họa, dẫn chứng minh cho BĐT Cauchy

Bài toán 1: (BĐT Cauchy) Cho n số thực không âm x1, , xn Chứng minh

raèng:

x1+ + xn ≥ nn

x1 xn

Chứng minh:

Bằng cách chuẩn hóa, ta giả sử x1 xn = chứng minh

x1+ + xn ≥ n Tất nhiên ta cần xét xi ≤ n, ∀i.

Xét: D = {x = (x1, , xn)|xi ∈ [0, n], x1 xn= 1} dễ thấy D đóng

bị chặn Xét Λ = {x0 = (1, 1, 1, , 1)}.

(49)

thì f(x) = x1 + + xn Xét T : D\Λ → D sau: Với x =

(x1, , xn) ∈ D\Λ, tồn xi 6= xj và ta đặt T (x) thu từ

x sau thay xi và xj trung bình nhân chúng, dễ thấy

f (x) − f (T (x)) = (xi

xj)2 > 0

Vậy ta áp dụng định lí để suy f(x) ≥ f(x0), ∀x ∈ D, nữa

daáu ” = ” xảy x = x0

Trong nhiều trường hợp, hàm f khơng đủ tốt ta chỉ có có điều kiện f(x) ≥ f(T (x)) Tất nhiên ta khơng thể áp dụng định lý Một đòi hỏi hợp lý phép biến đổi T phải đủ tốt để bù lại (nhớ phép biến đổi T ta chọn) Điều đưa đến:

Định lý 3: Cho:

D tập đóng, bị chận Rn, Λ tập đóng D

• T : D → D là phép biến đổi cho tồn hàm số h liên tục

D → R thỏa mãn: h(T (x)) < h(x),∀x ∈ D\Λ.

• f : D → R là hàm số liên tục thỏa mãn f(x) ≥ f(T (x))∀x ∈ D. Thì ta có GTNN f D GTNN f Λ, nghóa là:

f (x) ≥ min

y∈Λ{f (y)}, ∀x ∈ D.

Mặc dù trường hợp riêng định lý tổng quát cuối bài, tầm quan trọng định lý nên dẫn chứng minh cho

Chứng minh:

Lấy y0 ∈ Λsao cho f(y0) =

y∈Λ{f (y)} Giả sử phản chứng tồn tại

z ∈ D sao cho f(z) < f(y0) Tất nhiên ta giả sử h(x) ≥ 0, ∀x ∈ D

(nếu không việc thay h h0= h − M, với M GTNN h D).

Chọn ε > đủ nhỏ ta có: f(z) + εh(z) < f(y0) Đặt g(x) := f(x) + εh(x),

∀x ∈ D Thì g : D → R liên tục, g(x) > g(T (x))∀x ∈ D\Λ vaø g(z) <

f (y0) ≤

y∈Λ{g(y)} Điều mâu thuẫn với định lý 2.

Sau hệ ấn tượng định lý Hệ 1: (SMV-Strongly Mixing Variables) Cho: • D ∈ Rn, D = {x = (x

1, , xn)|xi ≥ α,

P

xi = ns = const} vaø s0 :=

(s, s, , s) ∈ D.

Phép biến đổi T : D → D sau: với phần tử a = (a1, , an) ∈ D,

(50)

ai6= aj, thay ai, aj bới trung bình cộng chúng

• f : D → R là hàm số liên tục thỏa mãn: f(a) ≥ f(T (a)), ∀a ∈ D. Khi đó: f(a) ≥ f(s0), ∀a ∈ D.

Chứng minh: Với phép biến đổi T vậy, ta chọn h(x) = Pn i=1x

2

i, ∀x =

(x1, , xn) ∈ D Áp dụng định lý (ở Λ = {s0})

*Nhận xét: Thông thường, áp dụng ta lấy ai, aj là max của

{a1, , an} Khi đó, chứng minh từ phần tử D, sau vơ

hạn lần lặp T thu (s, s, , s), sử dụng tính liên tục f ta cũng thu kết luận

Riêng trường hợp (min max) khơng thiết thay

ai, aj trung bình cộng mà tổng qt hơn:

Hệ 2: Cho:

• D ∈ Rn đóng bị chặn Gọi Λ tập hợp phần tử D có dạng (s, s, , s), giả sử Λ khác rỗng.

Phép biến đổi T : D → D sau: với phần tử a = (a1, , an) ∈ D\Λ,

ta chọn số i 6= j cho ai, aj là max {a1, , an}, sau

đó thay ai, aj bởi α, β ∈ (ai, aj)

• f : D → R là hàm số liên tục thỏa mãn: f(a) ≥ f(T (a)), ∀a ∈ D. Khi đó:

f (x) ≥ min

y∈Λ{f (y)}, ∀x ∈ D.

Chứng minh: Một cách tự nhiên, ta hi vọng vào hàm

h(a) = max{a1, , an} − min{a1, , an}, ∀a = (a1, , an) ∈ D

Tuy nhiên, ta khơng có h(a) > h(T (a)), ∀a ∈ D\Λ Đó trong n số a1, , an có thể có nhiều số max hay của

{a1, , an} Nhưng ta việc thay T T= Tn (Tk nghĩa lặp lại T

với k lần) h(a) > h(T(a)), ∀a ∈ D\Λ ta áp dụng định lý

Tuy nhiên, phép biến đổi T khơng đủ, ví dụ ta chưa biết xác dồn biến biên hay tâm Do đó, định lý mở rộng thành định lý dồn biến tổng quát sau

Định lý 4: (GMV − General Mixing Variables) Cho:

D tập đóng, bị chận Rn, Λ tập đóng D

• Tj : D → D là phép biến đổi cho tồn hàm số hj liên tục

(51)

• f : D → R liên tục thỏa mãn f(x) ≥ min

j∈{1, ,k}{f (Tj(x))}, ∀x ∈ D.

Thì f(x) ≥ min

y∈Λ{f (y)}, ∀x ∈ D.

Ta sử dụng lại chứng minh định lý với cải tiến nhỏ Chứng minh:

Laáy y0 ∈ Λsao cho f(y0) =

y∈Λ{f (y)} Giả sử phản chứng tồn tại

z ∈ D sao cho f(z) < f(y0) Tất nhiên ta giả sử hj(x) ≥ 0, ∀x ∈

D, ∀j = 1, , k Chọn ε > đủ nhỏ ta có: f(z)+εhj(z) < f (y0), ∀j = 1, , k.

Đặt gj(x) := f (x) + εhj(x), ∀x ∈ D, ∀j = 1, , k.

Đặt g(x) = min{g1(x), , gk(x)}, ∀x ∈ D Thì g : D → R liên tục,

g(x) > g(T (x))∀x ∈ D\Λ vaø g(z) < f(y0) ≤

y∈Λ{g(y)} Điều mâu

thuẫn với định lý

*Chi chú: Ta sử dụng kết gj là hàm liên tục g =

min{g1, , gk} hàm liên tục Tất nhiên ta cần chứng minh

với k = 2, trường hợp cần để ý min{g1, g2} =

2(g1+ g2− |g1− g2|) Còn kiện g(x) > g(T (x)), ∀x ∈ D\Λ rõ ràng,

vì uj > vj, ∀j = 1, , k thì min{u1, , uk} > min{u1, , uk}

Các bạn thân mến, hình thức phát biểu ngắn gọn GMV có tầm ứng dụng rộng rãi Cứ (hay vài) phép biến đổi T thích hợp ta lại có định lý dồn biến Chúng tơi kết thúc mục hệ GMV, mà xem mở rộng SMV Hệ Cũng xin lưu ý kết có tên SMV UMV tổng quát so với định lý tên mà chúng tơi dẫn ban đầu

Hệ 3: (UMV − Undefined Mixing Variables) Cho:

• D ⊂ {x = (x1, , xn) ∈ Rn|xi ≥ 0, ∀i = 1, , n}, D đóng bị chặn Gọi

Λ là tập hợp phần tử D có t thành phần n − t thành phần (t ≥ 0).

2 phép biến đổi T1, T2 : D → D như sau: với phần tử a = (a1, , an) ∈

D\Λ, chọn số i 6= j cho ai = min{at > 0, t = 1, , n} vaø

aj = max{a1, , an}, sau thay ai, aj bới α, β ∈ (ai, aj) (ứng với T1)

α0 < a

i < aj < β0 (ứng với T2)

• f : D → R liên tục thỏa mãn: f(a) ≥ min{f(T1(a)), f (T1(a))}, ∀a ∈ D.

Thì

f (x) ≥ min

(52)

Chứng minh:

Choïn h1(a) = max{a1, , an} − min{a1, , an} vaø h2(a) = −h1(a),

∀a = (a1, , an) ∈ D Tương tự hệ 2, ta thay T1 bởi T1∗ = T

n

1 vaø

T2∗ = T

n

2 để có: h1(a) > h1(T1∗(a)), h2(a) > h2(T2∗(a)), ∀a = (a1, , an) ∈ D.

Áp dụng GMV ta có điều phải chứng minh

9 Nhìn lại.

Các bạn thân mến, có lẽ lúc tạm dừng để nhìn lại hành trình vừa qua Như chúng tơi nói $6, dồn biến biết đến từ sớm thông qua hàm lồi dẫn đến kết tuyệt đẹp BĐT Jensen xem tiêu chuẩn để dồn biến tâm cách toàn cục Về kết này, bạn tìm đọc cách đầy đủ "Bất đẳng thức" tiếng nhà toán học Hardy − P olya − Littewood. Trong trường hợp biến, có lẽ quen thuộc với bạn đọc BĐT lượng giác, chẳng hạn như:

sinA + sinB + sinC ≤

2 (1)

cosA + cosB + cosC ≤

2 (2) với A, B, C cạnh tam giác.

BĐT (1) thu cách áp dụng BĐT Jensen cho hàm lồi BĐT (2) tinh tế hơn, hàm f(x) = −cosx có f00

(x) = cosx nên lồi [0, π] Do ta khơng thể áp dụng BĐT Jensen cho biến A,B,C Tuy nhiên, ta giả sử A ≤ B ≤ C A, B ∈ [0, π] nên ta dồn biến A, B Sau tốn cịn một biến trở nên đơn giản

(53)

Các bạn thân mến, dành mục để khảo sát vấn đề dồn biến cho BĐT biến để bạn nắm tư tưởng phương pháp, liệt kê tất kĩ thuật cần thiết Chẳng hạn dồn biến BĐT lượng giác với BĐT tuyệt đẹp Jackgarfulkel (xem phần tập) thú vị Tuy nhiên nghĩ trình bày tất nhàm chán vơ vị, nắm tư tưởng bạn áp dụng vơ vàn trường hợp khác

Đọc xong phần BĐT biến, có lẽ bạn đọc có cảm giác BĐT chuyển trường hợp biến biến đạt giá trị biên Phải nói điều cho hầu hết BĐT mà gặp Tuy nhiên, sau cung cấp cho bạn ví dụ nằm ngồi "thơng lệ" Trong ví dụ này, chí BĐT xét đa thức đối xứng biến Ví dụ lấy từ ý tưởng anh Bùi Việt Anh

Bài toán Cho a, b, c ≥ Khi BĐT:

(a3+ b3+ c3− 6abc)2 + ((a + b + c)3− 36abc)2 ≥

chỉ xảy dấu "=" trường hợp (a, b, c) = (t, 2t, 3t), t ≥ (và hoán vị)

Bạn đọc tự kiểm tra điều

Như vậy, bạn yên tâm phương pháp dồn biến có ý nghóa

Với tốn biến thơng thường phải thực lần động tác dồn biến nên phức tạp Trong trường hợp n biến tổng quát việc dồn biến trở nên khó khăn Ngồi BĐT Jensen cho phép dồn lúc n biến (nhưng đáng tiếc, giải lượng nhỏ BĐT) gần ta khơng có cơng cụ khác Trong trường hợp này, thông thường quy nạp ý hay Chúng dẫn ví dụ cho thấy tinh tế chứng minh quy nạp BĐT

Bài toán (Phạm Kim Hùng) Cho n số thực dương a1, a2, , an có tích

bằng Chứng minh với k > thì:

(1 + a1)k

+

(1 + a2)k

+ + (1 + an)k

≥ min{1, n 2k}

(54)

làm việc với toán tổng quát hơn:

"Cho n số thực dương a1, a2, , an có tích s ≥ Chứng minh với

mọi k > thì: (1 + a1)k

+

(1 + a2)k

+ + (1 + an)k

≥ min{1, n +√ns}.”

Với toán tổng quát lại chứng minh quy nạp Thật vậy, xét toán với n số, ta giả sử an = min{a1, , an} Khi

đó áp dụng giả thiết quy nạp cho (n − 1) số a1, a2, , an−1 có tích ≥ 1, ta

đưa toán biến Cơng việc cịn lại khảo sát hàm biến

Một kĩ thuật khác để đưa BĐT n biến biến dồn biến về giá trị trung binh $7 Như ra, ý tưởng cách dồn dựa cách dồn biến giá trị trung bình cho hàm lồi Đây cách dồn biến tốt có tính hữu hạn Tuy nhiên, áp dụng cho cực trị đạt tâm

Bây ta phải đối mặt với khả cực trị đạt tâm biên Rõ ràng khả dồn biến khơng cao Do hi vọng vào điều tốt có cách dồn biến toàn cục, BĐT Jensen Với mục tiêu đó, định lý tuyệt đẹp phải kể đến định lý SMV (dồn biến mạnh) UMV (dồn biến không xác định) Hai định lý nói "anh em song sinh" SMV dùng để "chuyên trị" BĐT cực trị đạt tâm, cải tiến đáng kể khơng cần dồn biến mà cần dồn biến lớn biến nhỏ UMV địi hỏi giả thiết đặt lên biến bất kì, nhiên cho phép ta dung hòa trường hợp cực trị đạt tâm biên dạng tổng quát Để cho hình thức đơn giản, định lý xét cho hàm đối xứng

Chúng quan sát kết nhận thấy không cần thiết việc tách rời trường hợp, tìm kết hội tụ đầy đủ ưu điểm định lý Tuy nhiên trường hợp riêng Hệ $8 Định lý GMV không đơn tổng quát định lý kể trên, mà mở chân trời với vơ vàn kiểu dồn biến Một điều kì lạ đòi hỏi: x = (x1, x2, , xn) chưa rơi vào

(55)

nhỏ nhất, dẫn đến cảm nhận rõ ràng n biến tiến giá trị trung bình, trường hợp bổ đề dãy số khơng cịn tác dụng Tuy nhiên, kết

Các bạn thân mến, bạn hành trình, mà chúng tơi chọn tốt khơng phải đầy đủ Có nhiều vấn đề chúng tơi khơng đưa ra, khơng trình bày kĩ, chúng tơi khơng coi trọng đầy đủ Cái mà coi trọng cố gắng để bạn thấy vấn đề cách nhanh chóng, rõ ràng hợp lý Hi vọng với tư tưởng mà khơi gợi bạn đủ cảm hứng khả để tiếp bước đường sáng tạo

Cuối cùng, muốn gửi lời cảm ơn đặc biệt tới anh Phan Thành Nam anh Phạm Kim Hùng, người có nhiều kết ý tưởng sử dụng Chúng xin chân thành cảm ơn tất tác giả tốn, nguồn trích dẫn, có thầy Phạm Văn Thuận − người cung cấp cho tài liệu dồn biến có giá trị

10 Bài tập.

Sau số tập dành cho bạn đọc Hi vọng bạn tìm nhiều niềm vui thử sức với chúng (ghi chú: Θ dễ, Φ trung bình, Ξ khó, Ψ cực khó)

Θ Bài tập 1: (Asian Pacific Math.2004) Giả sử a, b, c số dương tùy ý. Chứng minh BĐT

(a2+ 2)(b2 + 2)(c2+ 2) ≥ 9(ab + bc + ca)

Θ Bài tập 2: (MOSP 2001) Chứng minh a, b, c số dương có tích ta có BĐT

(a + b)(b + c)(c + a) ≥ 4(a + b + c − 1)

Θ Bài tập 3: Cho a, b, c không âm thỏa mãn a2+ b2+ c2 = Chứng minh

(56)

Θ Bài tập 4: (Huỳnh Tấn Châu) Cho x, y, z ≥ x + y + z = 1.Chứng minh rằng:

x3+ y3+ z3+ 6xyz ≥

Θ Bài tập 5: Chứng minh x, y, z số thực không âm thỏa mãn điều kiện x2

+ y2+ z2 = ta có BĐT:

7(xy + yz + zx) ≤ 12 + 9xyz

Φ Bài tập 6: (Chọn đội tuyển Việt Nam 1996) Cho a, b, c số thực bất kì, chứng minh rằng:

F (a, b, c) = (a + b)4+ (b + c)4+ (c + a)4− 7(a

4

+ b4+ c4) ≥

Φ Bài tập 7: (Phạm Văn Thuận−Zhao Bin) Giả sử x, y, z ba số thực khơng âm có nhiều số Chứng minh

1

x3+ y3 +

1

y3+ z3 +

1

z3+ x3 ≥

20 (a + b + c)3

Φ Bài tập (Phạm Kim Hùng) Chứng minh với số thực a, b, c không âm ta ln có BĐT:

1 √

4a2+ bc+

1 √

4b2+ ca+

1 √

4c2+ ab

4

a + b + c

Φ Bài tập 9: (Murray Klamkin) Chứng minh với số thực không âm a, b, c có tổng 2, thì

(a2+ ab + b2)(b2+ bc + c2)(c2+ ca + a2) ≤

Ξ Bài tập 10: (Tổng quát RMO2000) Cho a, b, c ≥ a + b + c = Tìm hằng số k > nhỏ cho BĐT sau đúng:

ak+ bk+ ck ≥ ab + bc + ca

(57)

Chứng minh rằng:

1

a2− bc + 1 +

1

b2− ca + 1 +

1

c2− ab + 1

Ξ Bài tập 12: (mathlinks) Cho a, b, c ≥ ab + bc + ca = Chứng minh rằng:

1 + a2b2

(a + b)2 +

1 + b2c2

(b + c)2 +

1 + c2a2

(c + a)2 ≥

5

Ξ Bài toán 13 Cho a, b, c ∈ [p, q] với < p ≤ q Tìm giá trị lớn của:

a b + c+

b c + a+

c a + b

Bài toán 14.(Jackgarfulkel) Cho tam giác nhọn ABC Chứng minh rằng: Φ a)

sinA

2 + sin

B

2 + sin

C

2 ≥

3(1 + sin

A 2sin B 2sin C 2) Φ b) cosA

2 + cos

B

2 + cos

C

2 ≥ √

3(1 + cos

A 2cos B 2cos C 2)

Φ Bài toán 15.(Jackgarfulkel) Cho tam giác ABC Chứng minh rằng:

cos(A − B

2 ) + cos(

B − C

2 ) + cos(

C − A

2 ) ≥ √

3(sinA + sinB + sinC)

ΞBài toán 16 (Phan Thành Nam) Cho ba sỗ thực x, y, z khơng âm có tổng Chứng minh

p

x + y2+py + z2 +√z + x2≥ 2

Ξ Bài tập 17 (Vasile Cirtoaje) Xét ba số thực không âm a, b, c thỏa điều kiện a2+ b2+ c2 = 1. Chứng minh rằng:

1 1 − ab+

1 1 − bc+

1 1 − ca

(58)

ΞBài tập 18: (Phan Thành Nam) Cho a, b, c ≥ thỏa mãn a + b + c = 1. Chứng minh rằng:

a) (VMEO1) r

a + (b − c)

2

12 +

r

b + (c − a)

2

12 +

r

c + (a − b)

2

12 ≤

b) p

a + k(b − c)2+pa + k(b − c)2+pa + k(b − c)2 ≤√3

Trong k = − √3

Ξ Bài tốn 19 (Phan Thành Việt) Cho tam giác ABC có độ dài cạnh là

BC = a, CA = b, AB = c Gọi p nửa chu vi tam giác ma, mb, mc

là độ dài ba đường trung tuyến tương ứng hạ từ A, B, C xuống cạnh đối diện Chứng minh rằng:

ma+ mb+ mc

r

3p2+

2[(a − b)

2+ (b − c)2+ (c − a)2]

Bài tập 20: (Phan Thành Nam) Cho x, y, z ∈ [−1, 1] x + y + z = 0. Chứng minh

Ξ a) p

1 + x + y2+p1 + y + z2 +

1 + z + x2 ≥ 3

Ψ b) r

1 + x + 9y

2+

r

1 + y + 9z

2+

r

1 + z + 9x

2 ≥ 3

Φ Bài tập 21 (Phạm Kim Hùng) Cho x, y, z, t ≥ x + y + z + t = 4. Chứng minh rằng:

(1 + 3x)(1 + 3y)(1 + 3z)(1 + 3t) ≤ 125 + 131xyzt

Φ Bài tập 22 (Bất đẳng thức Tukervici) Với số thực dương a, b, c, d thì

(59)

ΦBài tập 23 (Phạm Văn Thuận− Nguyễn Anh Tuấn) Xét số thực a, b, c, d thỏa mãn a2

+ b2+ c2+ d2 = 1.Chứng minh rằng

1 1 − ab+

1 1 − bc+

1 1 − cd +

1 1 − da +

1 1 − db +

1

1 − ca

Ξ Bài tập 24 (Phạm Kim Hùng) Cho số thực không âm a, b, c, d, k có tổng Chứng minh rằng

(abc)k + (bcd)k + (cda)k+ (dab)k ≤ max{4, (4 3)

3k

}

Ξ Bài tập 25 (Phan Thành Nam)

Cho số thực x, y, z, t thỏa: max{xy, yz, zt, tx} ≥ Chứng minh rằng p

1 − xy + y2+p1 − yz + z2+√1 − zt + t2+√1 − tx + x2

≥p16 + (x − y + z − t)2

Ψ Bài tập 26 (Phan Thành Nam) Cho số thực x, y, z, t ∈ [−1, 1] thỏa mãn x + y + z + t = Chứng minh rằng

p

1 + x + y2 +p1 + y + z2+

1 + z + t2 +

1 + t + x2 ≥ 4

(*Ghi chú: Bài xuất phát từ trường hợp ba số 20a, dĩ nhiên khó nhiều BĐT tương tự với số khơng cịn nữa)

Φ Bài tập 27 (Vasile Cirtoaje) Chứng minh a1, a2, , an khơng

âm có tổng n

(n − 1)(a21+ a

2+ + a

n) + na1a2 an≥ n2

Ξ Bài tập 28: (Phạm Kim Hùng) Giả sử a1, a2, , an số thực khơng

âm có tổng n Tìm gtnn biểu thức

S = a21+ a

2+ + a

n+ a1a2 an(

1

a1

+

a2

+ +

an

(60)

Ξ bài tập 29 Tìm số dương km tốt để BĐT sau với

mọi dãy số thực không âm x1, x2, , xn có tổng n

(1 + mx1)(1 + mx2) (1 + mxn) ≤ (m + 1)n+ km(x1x2 xn− 1)

trong m số dương bất kì.

Bài tốn 30 (Phan Thành Việt) Cho a1, a2, , an, s, k số thực dương

thỏa mãn: a1a2 an= sn và n − = (1+s)n k Xét BĐT:

1 (1 + a1)k

+

(1 + a2)k

+ + (1 + an)k

≤ n − 1

Θ a) Chứng minh BĐT nói chung không

Ngày đăng: 31/12/2020, 13:07

Tài liệu cùng người dùng

  • Đang cập nhật ...

Tài liệu liên quan